Сила тока через мощность и напряжение: Расчёт величины тока по мощности и напряжению

Содержание

Понятие силы тока, мощности, напряжения

При рассмотрении понятий силы тока, напряжения, мощности, нужно осознавать то, что все эти три параметра неразрывно связанны.
 

Мощность – это отношение производимой за определенный отрезок времени работы к данному отрезку времени. Единицей измерения является Ватт. 1 киловатт равен 1000 ватт.
 

Сила тока – это устремленное перемещение заряженных частиц. Выражает количество заряда, что пробегает сквозь разрез проводника за единицу времени. Единицей измерения является Ампер.
 

Напряжение – это отношение работы, выполненной электрическим полем для перенесения заряда, к значению переносимого заряда на полосе цепи. Этот параметр выражает работу, проделанную полем для передачи заряда между двумя точками. Единицей измерения является Вольт.
 

Можно провести аналогию описываемых параметров “силы”, “мощности” и “напряжения” с течением воды, где число ампер (сила тока) – это объем воды, пробегающей за единицу времени (обусловливается расходованием электричества, иными словами зависит от того, насколько сильно открыт кран), количество ватт (мощность) – это, к примеру, действие по приведению в движение лопастей турбины (давление перемноженное на силу тока), а значение вольт (напряжение) – это напор воды в трубопроводе. Стало быть, на блоке питания принципиальной является мощность, то есть, выдержит ли прибор, на батарейке – напряжение, потянет ли пользователь. На устройствах для электросети с установленным напряжением (220 вольт у нас) максимум силы тока, при перемножении этого показателя на значение напряжения, параллельно является и максимумом мощности.

 

Как же вычислить мощность с помощью величины напряжения и силы тока?
 

«Мощность» = «Сила тока» (Амперы) умножить на «Напряжение» (Ватты).
 

Как провести расчет силы тока по мощности и напряжению?
 

Исходя из предыдущей формулы, можем найти значение силы тока:
 

«Сила тока» = «Мощность» (Вольт-ампер) разделить на «Напряжение» (Ватты).
 

Существует еще пара значимых факторов, ежели речь идет об электричестве:
 

— Типичные розетки предусмотрены для силы тока числом 16 ампер. Так, как напряжение в электросети 220 вольт, значит, граничная мощность равна: 16 ампер умножить на 220 вольт = 3520 ватт (3,52 киловатт).

— В производстве розеток, в основном, используют 16-амперные автоматы. Из этого следует, что, когда на линии электропередачи с 16-амперным автоматом сила тока превысит 16 ампер (либо мощность возрастет свыше 3,52 киловатт), прибор автоматически вырубится. 

 

В частности, если в вашем доме проведена индивидуальная линия для кухонных розеток, то во время подсоединения к данной линии сразу двух электрообогревателей, с мощностью обоих в 2 киловатта, автомат разъединит электрическую цепь.

Зависимость мощности от силы тока, формула мощности, физический смысл

Первое упоминание об электричестве встречается в опытах древнегреческого философа Фалеса. Именно он первым обнаружил, что предметы при трении притягиваются. Одноименный термин был введен в начале 17-го века английским физиком Гилбертом, после опытов, проведенных с магнитами. Отцом же науки об электричестве считается французский ученый Кулон – именно после открытия закона, получившего его имя, электротехника начала свою победную поступь, которая продолжается до сих пор. Этот закон утверждает, что два точечных заряда в безвоздушной среде взаимодействуют с силой, прямо пропорциональной их модулям и обратно – расстоянию между ними, возведенному в квадрат.

Выясним, что же представляет собой понятие электричество?

Если коротко, то это – направленное движение потока заряженных частиц. Тела, через которые они проходят, называются проводниками. Каждый проводник имеет определенное сопротивление электрическому току, которое раз

И, перед тем, как перейти к основным законам, несколько слов о заряженных частицах: они бывают, условно говоря, положительными и отрицательными. Одноименные заряды отталкиваются, а разноименные – притягиваются.

А теперь, перейдем к главному.

Основа-основ науки об электричестве – закон Ома.

Эксперимент, который провел этот немецкий физик, привел его к следующему убеждению: сила тока I, проходящего через металлический проводник, пропорциональна напряжению на его концах, или I = U/R

Здесь напряжением называется разность, образно говоря, «давлений», созданных двумя точками электрической цепи. Измеряют его в вольтах. Электрический ток представляет собой число электронов, которые пропускает участок электрической цепи и измеряется в амперах. Сопротивлением считается свойство цепи помешать этому движению. В честь упомянутого физика, его измеряют в омах. Иначе говоря, проводник, через который проходит ток в 1 ампер при напряжении в 1 вольт, обладает сопротивлением в 1 ом.

Вся остальная электротехника «пляшет» от этого.

О мощности электрического тока

В физике мощностью считают скорость выполнения работы. Неважно, какой. Чем эта операция проводится быстрее, тем большей считается мощность того, кто ее исполняет, будь то человек, механическое устройство или что-то еще.

Так же и в случае с электрическим током: ее мощность представляет собой отношение работы, произведенной движущимися электрическими зарядами к промежутку времени, которое для этого понадобилось.

Проще говоря, для того, чтобы получить электрическую мощность в 1 ватт, когда источник тока имеет напряжение 1 вольт, необходимо пропустить через проводник ток в 1 ампер. Другими словами, мощность (P) можно посчитать, перемножив друг на друга электрическое напряжение и ток:

P = U*I.

Запомнив эту нехитрую формулу, на практике можно рассчитать мощность. Например, если известны значения тока и сопротивления, а о напряжении сведений нет, можем воспользоваться законом Ома, подставив в формулу вместо него I*R. Получится, что мощность равна квадрату электрического тока, помноженному на сопротивление.

Этот закон точно так же придет на помощь, если известны величины напряжения и сопротивления. В этом случае подставив вместо значения тока I = U/R, получим значение мощности, равное квадрату напряжения, поделенному на сопротивление.

Вот так – ничего сложного!

Сила тока и мощность тока. Simpleinfo – все сложное простыми словами!

07 Сентября 2017

7667

Подведем итоги по разделу. Обратим внимание на некоторые важные вещи и еще разберем пройденный материал.

1.В какую сторону течет ток?

Если вы обратили внимание, во всех предыдущих статьях, направление тока обозначено от (-) к (+), то есть с отрицательного полюса к положительному. Но в статье про закон Ома, мы указали с положительного полюса к отрицательному. В статье Электрическая проводимость мы выяснили, что носителем заряда являются отрицательно заряженные частицы, под воздействие поля происходит упорядоченное движение отрицательно заряженных частиц.

Таким образом направление движения тока с отрицательного полюса к положительному. Но в схематике (при разборе схем) и в быту используется направление от положительного к отрицательному. Как я понимаю это пришло с древности, пока точно не понимали, как движутся частицы.

наведите или кликните мышкой, для анимации

наведите или кликните мышкой, для анимации

Мы же, при разборе радиоэлементов, чтобы понять, как они работают будем использовать с отрицательного к положительному. А при разборе схем, с положительного полюса к отрицательному.  

2. Более простой разбор электрической цепи. Сколько потребляет нагрузка?

Мы теперь знаем, что такое замкнутая электрическая цепь. И как течет по нему ток. Также выяснили, что в цепи существует определенная сила тока, напряжение тока, сопротивление нагрузки или нагрузок, а также возникает выработка мощности. Теперь на практике выясним более подробнее.

Нужно запомнить, что чаще всего в электрической цепи, мы можем изменять напряжение тока и сопротивление нагрузки или нагрузок. К примеру, если у нас регулируемый источник питания, мы можем установить регулятор напряжения к отметке 5 В или 12 В. Если используются батарейки, можем взять 2 “пальчиковых” батарейки, это 3 В. Либо можем использовать 3 батарейки, таким образом уже будет 4,5 В. Что касается нагрузки, мы можем подключить 1 лампу накаливания или 2 и т.д., что приведет к изменению общего сопротивления нагрузки. А сила тока будет подстраиваться согласно закону Ома.

Силу тока нужно представлять себе так: показатель силы тока в цепи — это “потребление” нагрузки. Чем больше сила тока в цепи, чем больше потребляется ток нагрузкой. Давайте рассмотрим на примере, если взять две одинаковые аккумуляторные батареи и присоединить к ним разные нагрузки. Быстрее сядет та батарея, в цепи которой было больше силы тока.

Теперь возникает вопрос, если, меняя нагрузку, мы можем менять “потребление” тока, то значит меняя напряжение, мы также можем повлиять на “потребление” тока, то есть на силу тока. Так и есть, если мы увеличим напряжение, увеличится и ток в нагрузке. Но тут необходимо быть осторожным, так как если слишком большой ток пройдет через нагрузку, он может его испортить, так же наоборот, если недостаток тока, то устройство может не работать или работать плохо.

3. Чем отличается сила тока от мощности тока?

Еще раз вспоминаем, что такое сила тока и мощность тока.  
Сила тока — это прохождение частиц за единицу времени, выше мы с вами представили силу тока, как «потребление» нагрузки. К примеру, чтобы зажечь лампочку нужно создать в цепи 0,2 Ампера силы тока. Еще проще говоря, какая нужна сила, чтобы совершить, какое-то действие. (Зажечь лапочку, крутить двигатель, греть электроплиту и т.д.)

Мощность тока – это работа, которая выполняется за единицу времени нагрузкой. То есть, когда вращается двигатель — он совершает работу, когда электроплита греет — он совершает работу, когда лампочка горит – он так же совершает работу. Получается сила тока нам дает возможность выполнить работу, как бы отдавая свою энергию в нагрузку, далее нагрузка совершает ту или иную работу. При этом чем мощнее нагрузка, тем больше нужны заряды, соответственно больше силы тока в цепи. Более мощные нагрузки, выполняют больше работы. К примеру мощные электродвигатели сильнее крутятся, мощные лампочки ярче горят.

Таким образом, сила тока это, потребление тока нагрузкой или необходимое количества тока, для получения выработки мощности нагрузки. Мощность тока, это работа нагрузки за единицу времени. Сила тока и мощность тока взаимосвязаны. Что бы не путаться в голове нужно держать две вещи:

  • 1. В источниках питания пишут, показатель силы тока, то есть, сколько он сможет отдать.
  • 2. В нагрузках, в электроприборах пишут потребление в мощностях, то есть сколько ему нужно.

наведите или кликните мышкой, для анимации

формула, расчёт силы тока, напряжения и сопротивления

Напряжение, сопротивление, ток и мощность.

Электричество само по себе невидимо, хотя от этого его опасность ничуть не меньше. Даже наоборот: как раз потому и опаснее. Ведь если бы мы его видели, как видим, например, воду, льющуюся из крана, то наверняка бы избежали множества неприятностей.

Вода. Вот она, водопроводная труба, и вот закрытый кран. Ничего не течет, не капает. Но мы точно знаем: внутри вода. И если система исправно работает, то вода эта там находится под давлением. 2, 3 атмосферы, или сколько там? Неважно. Но давление там есть, иначе система бы не работала. Где-то гудят насосы, гонят воду в систему, создают это самое давление.

А вот наш провод электрический. Где-то далеко, на другом конце тоже гудят генераторы, вырабатывают электричество. И в проводе от этого тоже давление. Нет-нет, не давление, конечно, тут в этом проводе напряжение. Оно тоже измеряется, но в своих единицах: в вольтах.

Давит в трубах на стенки вода, никуда не двигаясь, ждет, когда найдется выход, чтобы ринуться туда мощным потоком. И в проводе молча ждет напряжение, когда замкнется выключатель, чтобы потоки электронов двинулись выполнять свое предназначение.

И вот открылся кран, потекла струя воды. По всей трубе течет, двигаясь от насоса к расходному крану. А как только замкнулись контакты выключателя, в проводах потекли электроны. Что это за движение? Это ток. Электроны текут. И это движение, этот ток тоже имеет свою единицу измерения: ампер.

И еще есть сопротивление. Для воды это, образно говоря, размер отверстия в выпускном кране. Чем больше отверстие, тем меньше сопротивление движению воды. В проводах почти также: чем больше сопротивление провода, тем меньше ток.

Вот, как-то так, если образно представлять себе основные характеристики электричества. А с точки зрения науки все строго: существует так называемый закон Ома. Гласит он следующим образом: I = U/R.
I – сила тока. Измеряется в амперах.
U – напряжение. Измеряется в вольтах.
R – сопротивление. Измеряется в омах.

Есть еще одно понятие – мощность, W. С ним тоже просто: W = U*I. Измеряется в ваттах.

Собственно, это вся необходимая и достаточная для нас теория. Из этих четырех единиц измерения в соответствии с вышеприведенными двумя формулами можно вывести некоторое множество других:

ЗадачаФормулаПример
1Узнать силу тока, если известны напряжение и сопротивление.I = U/RI = 220 в / 500 ом = 0.44 а.
2Узнать мощность, если известны ток и напряжение.W = U*IW = 220 в * 0.44 а = 96.8 вт.
3Узнать сопротивление, если известны напряжение и ток.R = U/IR = 220 в / 0.44 а = 500 ом.
4Узнать напряжение, если известны ток и сопротивление.U = I*RU = 0.44 а * 500 ом = 220 в.
5Узнать мощность, если известны ток и сопротивление.W = I 2 *RW = 0.44 а * 0.44 а * 500 ом = 96.8 вт.
6Узнать мощность, если известны напряжение и сопротивление.W = U 2 /RW = 220 в * 220 в / 500 ом = 96.8 вт.
7Узнать силу тока, если известны мощность и напряжение.I = W/UI = 96.8 вт / 220 в = 0,44 а.
8Узнать напряжение, если известны мощность и ток.U = W/IU = 96.8 вт / 0.44 а = 220 в.
9Узнать сопротивление, если известны мощность и напряжение.R = U 2 /WR = 220 в * 220 в / 96.8 вт = 500 ом.
10Узнать сопротивление, если известны мощность и ток.R = W/I 2R = 96.8 вт / (0,44 а * 0,44 а) = 500 ом.

Ты скажешь: – Зачем мне это все надо? Формулы, цифры. Я ж не собираюсь заниматься расчетами.

А я так отвечу: – Перечитай предыдущую статью Электроснабжение. Основы.. Как можно быть уверенным, не зная простейших истин и расчетов? Хотя, собственно, в бытовом практическом плане наиболее интересна только формула 7, где определяется сила тока при известных напряжении и мощности. Как правило, эти 2 величины известны, а результат (сила тока) безусловно необходим для определения допустимого сечения провода и для выбора защиты.

Есть еще одно обстоятельство, о котором следует упомянуть в контексте этой статьи. В электроэнергетике используется так называемый “переменный” ток. То есть, те самые электроны движутся в проводах не всегда в одном направлении, они постоянно меняют его: вперед-назад-вперед-назад. И эта смена направления движения – 100 раз в секунду.

Погоди, но ведь везде говорится, что частота 50 герц! Да, именно так и есть. Частота измеряется в количестве периодов за секунду, но в каждом периоде ток меняет свое направление дважды. Иначе сказать, в одном периоде две вершины, которые характеризуют максимальное значение тока (положительное и отрицательное), и именно в этих вершинах происходит смена направления.

Не будем вдаваться в подробности более глубоко, но все же: почему именно переменный, а не постоянный ток?

Вся проблема в передаче электроэнергии на большие расстояния. Тут как раз вступает в силу неумолимый закон Ома. При больших нагрузках, если напряжение 220 вольт, сила тока может быть очень большой. Для передачи электроэнергии с таким током потребуются провода очень большого сечения.

Выход здесь только один: поднять напряжение. Седьмая формула говорит: I = W/U. Совершенно очевидно, что если мы будем подавать напряжение не 220 вольт, а 220 тысяч вольт, то сила тока уменьшится в тысячу раз. А это значит, что сечение проводов можно взять намного меньше.

В этой статье уже не раз я обмолвился о зависимости сечения проводника от силы протекаемого тока. О том, как определить допустимое значение, узнаем в следующей статье Допустимый длительный ток..

Как найти мощность: формула, расчёт силы тока, напряжения и сопротивления

Причиной написания данной статьи явилась не сложность этих формул, а то, что в ходе проектирования и разработки каких-либо схем часто приходится перебирать ряд значений чтобы выйти на требуемые параметры или сбалансировать схему. Данная статья и калькулятор в ней позволит упростить этот подбор и ускорить процесс реализации задуманного. Также в конце статьи приведу несколько методик для запоминания основной формулы закона Ома. Эта информация будет полезна начинающим. Формула хоть и простая, но иногда есть замешательство, где и какой параметр должен стоять, особенно это бывает поначалу.

В радиоэлектронике и электротехнике закон Ома и формула расчёта мощности используются чаше чем какие-либо из всех остальных формул. Они определяют жесткую взаимосвязь между четырьмя самыми ходовыми электрическими величинами: током, напряжением, сопротивлением и мощностью.

Закон Ома. Эту взаимосвязь выявил и доказал Георг Симон Ом в 1826 году. Для участка цепи она звучит так: сила тока прямо пропорциональна напряжению, и обратно пропорциональна сопротивлению

Так записывается основная формула:

Путем преобразования основной формулы можно найти и другие две величины:

Мощность. Её определение звучит так: мощностью называется произведение мгновенных значений напряжения и силы тока на каком-либо участке электрической цепи.

Формула мгновенной электрической мощности:

Ниже приведён онлайн калькулятор для расчёта закона Ома и Мощности. Данный калькулятор позволяет определить взаимосвязь между четырьмя электрическими величинами: током, напряжением, сопротивлением и мощностью. Для этого достаточно ввести любые две величины. Стрелками «вверх-вниз» можно с шагом в единицу менять введённое значение. Размерность величин тоже можно выбрать. Также для удобства подбора параметров, калькулятор позволяет фиксировать до десяти ранее выполненных расчётов с теми размерностями с которыми выполнялись сами расчёты.

Когда мы учились в радиотехническом техникуме, то приходилось запоминать очень много всякой всячины. И чтобы проще было запомнить, для закона Ома есть три шпаргалки. Вот какими методиками мы пользовались.

Первая – мнемоническое правило. Если из формулы закона Ома выразить сопротивление, то R = рюмка.

Вторая – метод треугольника. Его ещё называют магический треугольник закона Ома.

Если оторвать величину, которую требуется найти, то в оставшейся части мы получим формулу для её нахождения.

Третья. Она больше является шпаргалкой, в которой объединены все основные формулы для четырёх электрических величин.

Пользоваться ею также просто, как и треугольником. Выбираем тот параметр, который хотим рассчитать, он находиться в малом кругу в центре и получаем по три формулы для его расчёта. Далее выбираем нужную.

Этот круг также, как и треугольник можно назвать магическим.

Как рассчитать мощность электрического тока?

Большинство бытовых приборов, подключаемых к сети, характеризуются таким параметром, как электрическая мощность устройства. С физической точки зрения мощность представляет собой количественное выражение совершаемой работы. Поэтому для оценки эффективности того или иного устройства вам необходимо знать нагрузку, которую он будет создавать в цепи. Далее мы рассмотрим особенности самого понятия и как найти мощность тока, обладая различными характеристиками самого устройства и электрической сети.

Понятие электрической мощности и способы ее расчета

С электротехнической точки зрения она представляет собой количественное выражение взаимодействия энергии с материалом проводников и элементами при протекании тока в электрической цепи. Из-за наличия электрического сопротивления во всех деталях, задействованных в проведения электротока, направленное движение заряженных частиц встречает препятствие на пути следования. Это и обуславливает столкновение носителей заряда, электроэнергия переходит в другие виды и выделяется в виде излучения, тепла или механической энергии в окружающее пространство. Преобразование одного вида в другой и есть потребляемая мощность прибора или участка электрической цепи.

В зависимости от параметров источника тока и напряжения мощность также имеет отличительные характеристики. В электротехнике обозначается S, P и Q, единица измерения согласно международной системы СИ – ватты. Вычислить мощность можно через различные параметры приборов и электрических приборов. Рассмотрим каждый из них более детально.

Через напряжение и ток

Наиболее актуальный способ, чтобы рассчитать мощность в цепях постоянного тока – это использование данных о силе тока и приложенного напряжения. Для этого вам необходимо использовать формулу расчета: P = U*I

  • P – активная мощность;
  • U – напряжение приложенное к участку цепи;
  • I — сила тока, протекающего через соответствующий участок.

Этот вариант подходит только для активной нагрузки, где постоянный ток не обеспечивает взаимодействия с реактивной составляющей цепи. Чтобы найти мощность вам нужно выполнить произведение силы тока на напряжение. Обе величины должны находиться в одних единицах измерения – Вольты и Амперы, тогда результат также получится в Ваттах. Можно использовать и другие способы кВ, кА, мВ, мА, мкВ, мкА и т.д., но и параметр мощности пропорционально изменит свой десятичный показатель.

Через напряжение и сопротивление

Для большинства электрических устройств известен такой параметр, как внутреннее сопротивление, которое принимается за константу на весь период их эксплуатации. Так как бытовые или промышленные единицы подключаются к источнику с известным номиналом напряжения, определять мощность достаточно просто. Активная мощность находится из предыдущего соотношения и закона Ома, согласно которого ток на участке прямо пропорционален величине приложенного напряжения и имеет обратную пропорциональность к сопротивлению:

I = U/R

Если выражение для вычисления токовой нагрузки подставить в предыдущую формулу, то получится такое выражение для определения мощности:

P = U*(U/R)=U 2 /R

  • P – величина нагрузки;
  • U – приложенная разность потенциалов;
  • R – сопротивление нагрузки.

Через ток и сопротивление

Бывает ситуация, когда разность потенциалов, приложенная к электрическому прибору, неизвестна или требует трудоемких вычислений, что не всегда удобно. Особенно актуален данный вопрос, если несколько устройств подключены последовательно и вам неизвестно, каким образом потребляемая электроэнергия распределяется между ними. Подход в определении здесь ничем не отличается от предыдущего способа, за основу берется базовое утверждение, что электрическая нагрузка рассчитывается как P = U×I, с той разницей, что напряжение нам не известно.

Поэтому ее мы также выведем из закона Ома, согласно которого нам известно, что падение напряжения на каком-либо отрезке линии или электроустановки прямо пропорционально току, протекающему по этому участку и сопротивлению отрезка цепи:

U=I*R

после того как выражение подставить в формулу мощности, получим:

P = (I*R)*I =I 2 *R

Как видите, мощность будет равна квадрату силы тока умноженной на сопротивление.

Полная мощность в цепи переменного тока

Сети переменного тока кардинально отличаются от постоянного тем, что изменение электрических величин, приводит к появлению не только активной, но и реактивной составляющей. В итоге суммарная мощность будет также состоять активной и реактивной энергии:

  • S – полная мощность
  • P – активная составляющая – возникает при взаимодействии электротока с активным сопротивлением;
  • Q – реактивная составляющая – возникает при взаимодействии электротока с реактивным сопротивлением.

Также составляющие вычисляются через тригонометрические функции, так:

P = U*I*cosφ

Q = U*I*sinφ

что активно используется в расчете электрических машин.

Рис. 1. Треугольник мощностей

Пример расчета полной мощности для электродвигателя

Отдельный интерес представляет собой нагрузка, подключенная к трехфазной сети, так как электрические величины, протекающие в ней, напрямую зависят от номинальной нагрузки каждой из фаз. Но для наглядности примера мы не будем рассматривать, как найти мощность несимметричного прибора, так как это довольно сложная задача, а приведем пример расчета трехфазного двигателя.

Особенность питания и асинхронной и синхронной электрической машины заключается в том, что на обмотки может подаваться и фазное и линейное напряжение. Тот или иной вариант, как правило, обуславливается способом соединения обмоток электродвигателя. Тогда мощность будет вычисляться по формуле:

В случае выполнения расчетов с линейным напряжением, чтобы найти мощность формула примет вид:

Активная и реактивная мощности будут вычисляться по аналогии с сетями переменного тока, как было рассмотрено ранее.

Теперь рассмотрим вычисления на примере конкретной электрической машины асинхронного типа. Следует отметить, что официальная производительность, указываемая в паспортных данных электродвигателя – это полезная мощность, которую двигатель может выдать при совершении оборотов вала. Однако полезная кардинально отличается от полной, которую можно вычислить за счет коэффициента мощности.

Рис. 2. Шильд электродвигателя

Как видите, для вычислений с шильда мы возьмем следующую информацию об электродвигателе:

  • полезная производительность – 3 кВт, а в переводе на систему измерения – 3000 Вт;
  • коэффициент полезного действия – 80%, а в пересчете для вычислений будем пользоваться показателем 0,8;
  • тригонометрическая функция соотношения активных и реактивных составляющих – 0,74%;
  • напряжение, при соединении обмоток треугольником составит 220 В;
  • сила тока при том же способе соединения – 13,3 А.

С таким перечнем характеристик можно воспользоваться несколькими способами:

S = 1,732*220*13,3 = 5067 Вт

Чтобы найти искомую величину, сначала определяем активную составляющую:

P = Pполезная / КПД = 3000/0.8 = 3750 Вт

Далее полную по способу деления активной на коэффициент cos φ:

S = P/cos φ = 3750/0.74 = 5067 Вт

Как видите, и в первом, и во втором случае искомая величина получилась одинакового значения.

Примеры задач

Для примера рассмотрим вычисление на участках электрической цепи с последовательным и параллельным соединением элементов. Первый вариант предусматривает ситуацию, когда все детали соединяются друг за другом от одного полюса источника питания до другого.

Рис. 3. Последовательная расчетная цепь

Как видите на рисунке, в качестве источника мы используем батарейку с номинальным напряжением 9 В и три резистора по 10, 20 и 30 Ом соответственно. Так как номинальный ток нам не известен, расчет произведем через напряжение и сопротивление:

P = U 2 /R = 81 / (10+20+30) = 1.35 Вт

Для параллельной схемы подключения возьмем в качестве примера участок цепи с двумя резисторами и одним источником тока:

Рис. 4. Параллельная схема подключения

Как видите, для удобства расчетов нам нужно привести параллельно подключенные резисторы к схеме замещения, из чего получится:

Тогда искомый номинал нагрузки мы можем узнать через значение тока и сопротивления:

Формула напряжения тока

Электротехника как область науки, занимающаяся использованием электроэнергии, в том числе ее получением, распределением и учетом, оперирует значениями тока, напряжения, мощности и сопротивления. Это основные величины. Кроме этого, имеется множество других характеристик и понятий, но в рамках данной статьи будут рассматриваться именно эти основополагающие понятия.

Электрический ток

Согласно определению, ток представляет собой упорядоченное движение заряженных частиц в среде. Такими частицами могут быть свободные электроны или ионы, частицы вещества, в которых число протонов в ядре не равно количеству электронов, то есть имеющие определенный заряд, положительный или отрицательный. Электроток может быть постоянный или переменный.

Электрическое напряжение

Электрическое напряжение – это разность потенциалов на противоположных участках цепи. Точное определение понятия подразумевает работу по переносу электрического заряда между участками цепи.

Сопротивление

Любой проводник в цепи препятствует прохождению через себя тока. Данная характеристика определяет такую физическую величину, как сопротивление. Исходя из величины сопротивления, все вещества относят к проводникам или изоляторам. Точная граница весьма расплывчата, поэтому при некоторых условиях некоторые вещества можно отнести как к изоляторам, так и к проводникам. Участок электросхемы может иметь элемент с определенным значением величины, который именуется резистор.

Мощность

Скорость преобразования, передачи и потребления электрической энергии определяется мощностью.

Взаимосвязь параметров электрической цепи

Все параметры любой электрической цепи строго взаимосвязаны, поэтому в любой момент времени можно точно определить величину любого из них, зная остальные.

К сведению. Основополагающий закон, по которому производится большинство расчетов, – закон Ома, согласно которому сила тока обратно пропорциональна его сопротивлению и прямо пропорциональна приложенной разности потенциалов.

Формула напряжения тока закона Ома выглядит следующим образом:

Так, цепь с большим напряжением пропускает больший ток, а при одинаковом напряжении ампераж будет больше там, где меньше сопротивление.

Принятые обозначения в формуле расчета напряжения и тока понятны во всем мире:

  • I – сила тока;
  • U – напряжение;
  • R – сопротивление.

Путем простейшего математического преобразования находится формула расчета сопротивления через силу тока и напряжение.

Кроме закона Ома, используется формула расчета мощности:

Символом P здесь обозначена мощность тока.

Любая схема может содержать участки, где имеется последовательное соединение, или есть элемент, подключенный параллельно. Расчеты при этом усложняются, но базовые формулы остаются одинаковыми.

Единицы измерения в формуле

Невозможно выполнять расчеты или измерения, не зная, какими величинами оперировать. Общепринятые обозначения, согласно международной системе измерения СИ:

  • Напряжение – Вольт. Обозначается символом В или V в англоязычной литературе;
  • Сила тока – Ампер. Обозначается символом А;
  • Электрическое сопротивление – Ом. Используется обозначение Ом или Ohm;
  • Электрическая мощность – Ватт. Обозначается как Вт или W.

Как работает закон в реальной жизни

Используя совместно формулу расчета мощности и закон Ома, можно производить вычисления, не зная одной из величин. Самый простой пример – для лампы накаливания известны только ее мощность и напряжение. Применяя приведенные выше формулы, можно легко определить параметры нити накаливания и ток через нее.

Сила тока формула через мощность:

Сопротивление:

Такой же результат можно найти из мощности, не прибегая к промежуточным расчетам:

Аналогично можно вычислить любую величину, зная только две из них. Для упрощения преобразований имеется мнемоническое отображение формул, позволяющее находить любые величины.

Внимательно посмотрев на формулы, можно заметить, что, если уменьшить напряжение на лампе в два раза, ожидаемая мощность не снизится аналогично в два раза, а в четыре, согласно формуле:

Это довольно распространенная ошибка среди далеких от электротехники людей, которые неправильно соотносят мощность и напряжение, а также их действие на остальные параметры.

Кстати. Сила тока, найденная через сопротивление и напряжение, справедлива как для постоянного, так и для переменного тока, если в ней не используются такие элементы, как конденсатор или индуктивность.

Облегчить расчеты можно, используя онлайн калькулятор.

Пример с обычной водой

Существуют вещества, которые можно отнести одновременно к проводникам и изоляторам. Самый простой пример – обыкновенная вода. Дистиллированная вода является хорошим изолятором, но наличие в ней практически любых примесей делает ее проводником. Особенно это относится к солям различных металлов. При растворении в воде соли диссоциируются на ионы, их наличие – прямой повод для возникновения тока. Чем больше концентрация солей, тем меньшим сопротивлением будет обладать вода.

Для наглядности можно взять дистиллированную воду для приготовления электролита для автомобильных аккумуляторных батарей. Опустив щупы омметра в воду, можно увидеть, что его показания велики. Добавление всего нескольких кристаллов поваренной соли через некоторое время вызывает резкое уменьшение сопротивления, которое будет тем меньше, чем больше соли перейдет в раствор.

По какой формуле определяется напряжение

Использование той или иной формулы напряжения электрического тока для вычисления зависит от того, какие величины известны:

  • Ток и сопротивление – U=I∙R;
  • Ток и мощность – U=P/I;
  • Мощность и сопротивление – U=√P∙R

Различные используемые величины

Кроме основных величин: вольт, ампер, ом, ватт, используют кратные, большие или меньшие. Для обозначений применяют соответствующие приставки:

  • Кило – 1000;
  • Мега – 1000000;
  • Гига – 1000000000;
  • Милли – 0.001.

Таким образом, получается:

  • Киловольт (кВ) – тысяча вольт;
  • Мегаватт (Мвт) – миллион ватт;
  • Миллиом (мОм) – одна тысячная Ом;
  • Гигаватт (ГВт) – тысяча мегаватт или миллиард ватт.

Как найти напряжение

Формула нахождения напряжения как разности потенциалов в электрическом поле:

U=ϕA-ϕB, где ϕAи ϕB – потенциалы в точках А и В, соответственно.

Также можно записать напряжение как работу по переносу единицы заряда из точки А в точку В в электрическом поле:

U=A/q, где q – величина заряда.

Работа тем больше, чем выше напряженность электрического поля Е, то есть сила, действующая на неподвижный заряд.

Потенциальную энергию заряда в электростатическом поле называют электростатический потенциал.

Гидравлическая аналогия

Чтобы легче усвоить законы электрических цепей, можно представить себе аналогию с гидравлической системой, в которой соединение насоса и трубопроводов образует замкнутую систему. Для этого нужны следующие соответствия:

  • Источник питания – насос;
  • Проводники – трубы;
  • Электроток – движение воды.

Без особых усилий становится понятнее, что чем меньше диаметр труб, тем медленнее по ним движется вода. Чем мощнее насос, тем большее количество воды он способен перекачать. При одинаковой мощности насоса уменьшение диаметра труб приведет к снижению потока воды.

Измерительные приборы

Для измерения параметров электрических цепей служат измерительные приборы:

Наиболее часто используется класс комбинированных устройств, в которых переключателем выбирается измеряемая величина – ампервольтомметры или авометры.

Типичные напряжения

Для стандартизации и возможности использования различного оборудования в быту и технике применяются электрические сети со стандартными значениями:

  • Бытовая сеть –220В;
  • Бортовая сеть автомобиля – 12 или 24В;
  • Батареи и аккумуляторы – 1.5, 3 или 9В.

Потенциал Гальвани

В электрохимии используется понятие потенциала Гальвани, который означает разность потенциала между различными фазами вещества, например, между электродом и электролитом, между электродами из разнородных металлов.

Видео

Особенности расчета мощности по току и напряжению

Чтобы электропроводка и все электрическое оборудование, которое имеется в доме, работало исправно и правильно, необходимо правильно сделать вычисление мощности по току и электронапряжению, поскольку при неправильно подобранных показателях может возникнуть короткое замыкание или возгорание. Как сделать расчёт потребляемой мощности по току и напряжению, как вычисляется сила тока, формула через мощность и напряжение и другое, далее.

Как узнать силу тока, зная мощность и напряжения

Чтобы ответить на вопрос, как определить ток, необходимо поделить электронапряжение на общее число ватт. При этом сделать все необходимые вычисления можно самостоятельно, а можно прибегнуть к специальному онлайн-калькулятору.

Узнать потребление электроэнергии по токовой силе резистора можно умножением первой на сопротивление, выражаемое в Омах. В итоге, получится значение, представленное в вольтах, перемноженных на ом. Получится ампер.

Обратите внимание! Если нет сопротивления, нужно поделить ваттный показатель на токовую энергию, то есть следует поделить ватты на амперы и получится значение электроэнергии в вольтах. Понять мощностное показание через величину электричества с электронапряжением, можно умножив соответствующие показания с устройства.

Формулы для расчета тока в трехфазной сети

Подсчитать токовую энергию в трехфазной сети сложно, поскольку вместе одной фазы есть три. К тому же, сложность заключается в использовании нескольких схем соединения. Трудность состоит в симметрии или ее отсутствии во время распределения нагрузки по фазам.

Для определения силы тока в трехфазной сети, нужно общее число ватт поделить на показатель 1,73, перемноженный на напряжение и косинус мощностного коэффициента, который отражает активную и реактивную составляющую сопротивления нагрузки. Что касается однофазной сети, то из выражения для подсчета убирается показатель 1,73. Остается формула I = P/(U*cos φ).

Как рассчитать ампераж

Ампераж является значением электротока, которое выражена в амперах. Рассчитать ампераж можно так: I=P/U.

Расчет потребляемой мощности

Электромощность является величиной, которая отвечает за факт скорости изменения или передачи электрической энергии. Есть полная и активная мощностная нагрузка, а также активная и реактивная. Полная вычисляется так: S = √ (P2 + Q2), где P является активной частью, а Q реактивной. Для нахождения потребляемого мощностного показателя необходимо знать число электротока, которое потребляется нагрузкой, а также питательное напряжение, которое выдается при помощи источника.

Что касается бытового определения потребляемой электрической энергии, необходимо вычислить общее количество ватт питания электрических приборов и паспортные данные номинальной силы электротока котла. Как правило, все электрические приборы работают с переменным током и напряжением в 220 вольт. Для вычисления тока проще всего воспользоваться амперметром. Зная первый и второй параметры, реально узнать величину потребляемой энергии.

Стоит указать, что измерить мощность через напряжение или сделать расчет мощности по сопротивлению и напряжению возможно не только формулой, но и прибором. Для этого можно воспользоваться мультиметром с токоизмерительными клещами или специализированным измерителем — ваттметром.

Обратите внимание! Оба работают по одному и тому же принципу, указанному в руководстве по их эксплуатации.

Мощность, ток и напряжение — три составляющие расчета проводки в доме. Узнать все необходимые параметры в любой сети просто при помощи формул, представленных выше. От этих значений будет зависеть исправность работы всей домашней электрики и безопасность ее владельца.


Как найти мощность тока — формулы с примерами расчетов

Определение

Мощность – это скалярная величина. В общем случае она равна отношению выполненной работы ко времени:

P=dA/dt

Простыми словами эта величина определяет, как быстро выполняется работа. Она может обозначаться не только буквой P, но и W или N, измеряется в Ваттах или киловаттах, что сокращенно пишется как Вт и кВт соответственно.

Электрическая мощность равна произведению тока на напряжение или:

P=UI

Как это связано с работой? U – это отношение работы по переносу единичного заряда, а I определяет, какой заряд прошёл через провод за единицу времени. В результате преобразований и получилась такая формула, с помощью которой можно найти мощность, зная силу тока и напряжение.

Формулы для расчётов цепи постоянного тока

Проще всего посчитать мощность для цепи постоянного тока. Если есть сила тока и напряжение, тогда нужно просто по формуле, приведенной выше, выполнить расчет:

P=UI

Но не всегда есть возможность найти мощность по току и напряжению. Если вам они не известны – вы можете определить P, зная сопротивление и напряжение:

P=U 2 /R

Также можно выполнить расчет, зная ток и сопротивление:

P=I 2 *R

Последними двумя формулами удобен расчёт мощности участка цепи, если вы знаете R элемента I или U, которое на нём падает.

Для переменного тока

Однако для электрической цепи переменного тока нужно учитывать полную, активную и реактивную, а также коэффициент мощности (соsФ). Подробнее все эти понятия мы рассматривали в этой статье: https://samelectrik.ru/chto-takoe-aktivnaya-reaktivnaya-i-polnaya-moshhnost.html.

Отметим лишь, что чтобы найти полную мощность в однофазной сети по току и напряжению нужно их перемножить:

S=UI

Результат получится в вольт-амперах, чтобы определить активную мощность (ватты), нужно S умножить на коэффициент cosФ. Его можно найти в технической документации на устройство.

P=UIcosФ

Для определения реактивной мощности (вольт-амперы реактивные) вместо cosФ используют sinФ.

Q=UIsinФ

Или выразить из этого выражения:

И отсюда вычислить искомую величину.

Найти мощность в трёхфазной сети также несложно, для определения S (полной) воспользуйтесь формулой расчета по току и фазному напряжению:

А зная Uлинейное:

1,73 или корень из 3 – эта величина используется для расчётов трёхфазных цепей.

Тогда по аналогии чтобы найти P активную:

Определить реактивную мощность можно:

На этом теоретические сведения заканчиваются и мы перейдём к практике.

Пример расчёта полной мощности для электродвигателя

Мощность у электродвигателей бывает полезная или механическая на валу и электрическая. Они отличаются на величину коэффициента полезного действия (КПД), эта информация обычно указана на шильдике электродвигателя.

Отсюда берём данные для расчета подключения в треугольник на Uлинейное 380 Вольт:

Тогда найти активную электрическую мощность можно по формуле:

P=Pна валу/n=160000/0,94=170213 Вт

Теперь можно найти S:

Именно её нужно найти и учитывать, подбирая кабель или трансформатор для электродвигателя. На этом расчёты окончены.

Расчет для параллельного и последовательного подключения

При расчете схемы электронного устройства часто нужно найти мощность, которая выделяется на отдельном элементе. Тогда нужно определить, какое напряжение падает на нём, если речь идёт о последовательном подключении, или какая сила тока протекает при параллельном включении, рассмотрим конкретные случаи.

Здесь Iобщий равен:

На каждом резисторе R1 и R2, так как их сопротивление одинаково, напряжение падает по:

И выделяется по:

Pна резисторе=UI=6*0,6=3,6 Ватта

Тогда при параллельном подключении в такой схеме:

Сначала ищем I в каждой ветви:

И выделяется на каждом по:

Или через общее сопротивление, тогда:

Все расчёты совпали, значит найденные значения верны.

Заключение

Как вы могли убедиться найти мощность цепи или её участка совсем несложно, неважно речь идёт о постоянке или переменке. Важнее правильно определить общее сопротивление, ток и напряжение. Кстати этих знаний уже достаточно для правильного определения параметров схемы и подбора элементов – на сколько ватт подбирать резисторы, сечения кабелей и трансформаторов. Также будьте внимательны при расчёте S полной при вычислении подкоренного выражения. Стоит добавить лишь то, что при оплате счетов за коммунальные услуги мы оплачиваем за киловатт-часы или кВт/ч, они равняются количеству мощности, потребленной за промежуток времени. Например, если вы подключили 2 киловаттный обогреватель на пол часа, то счётчик намотает 1 кВт/ч, а за час – 2 кВт/ч и так далее по аналогии.

Напоследок рекомендуем просмотреть полезное видео по теме статьи:

Также читают:

Работа и мощность тока — урок. Физика, 8 класс.

При прохождении тока в цепи электрическое поле совершает работу по перемещению заряда. В этом случае работу электрического поля называют работой электрического тока.

При прохождении заряда \(q\) по участку цепи электрическое поле будет совершать работу: \(A=q\cdot U\), где \(U\) — напряжение электрического поля, \(A\) — работа, совершаемая силами электрического поля по перемещению заряда \(q\) из одной точки в другую.

Для выражения любой из этих величин можно использовать приведённые ниже рисунки.


Количество заряда, прошедшее по участку цепи, пропорционально силе тока и времени прохождения заряда: q=I⋅t.

Работа электрического тока на участке цепи пропорциональна напряжению на её концах и количеству заряда, проходящего по этому участку: A=U⋅q.

Работа электрического тока на участке цепи пропорциональна силе тока, времени прохождения заряда и напряжению на концах участка цепи: A=U⋅I⋅t.

Чтобы выразить любую из величин из данной формулы, можно воспользоваться рисунком.

 

 

Единицы измерения величин:

работа электрического тока \([A]=1\) Дж;

напряжение на участке цепи \([U]=1\) В;

сила тока, проходящего по участку \([I]=1\) А;

время прохождения заряда (тока) \([t]=1\) с.

Для измерения работы электрического тока нужны вольтметр, амперметр и часы. Например, для определения работы, которую совершает электрический ток, проходя по спирали лампы накаливания, необходимо собрать цепь, изображённую на рисунке. Вольтметром измеряется напряжение на лампе, амперметром — сила тока в ней. А при помощи часов (секундомера) засекается время горения лампы.


 

Например:

 

I = 1,2 АU = 5 Вt = 1,5 мин = 90 сА = U⋅I⋅t = 5⋅1,2⋅90 = 540 Дж 

 

Обрати внимание!

Работа чаще всего выражается в килоджоулях или мегаджоулях.

\(1\) кДж = 1000 Дж или \(1\) Дж = \(0,001\) кДж;
\(1\) МДж = 1000000 Дж или \(1\) Дж = \(0,000001\) МДж.

На практике работу электрического тока измеряют специальными приборами — счётчиками электрической энергии. Их можно увидеть как в каждом частном доме, так в каждом подъезде многоквартирного дома.

 

Механическая мощность численно равна работе, совершённой телом в единицу времени: N = Аt.  Чтобы найти мощность электрического тока, надо поступить точно также, т.е. работу тока, A=U⋅I⋅t, разделить на время.

Мощность электрического тока обозначают буквой \(Р\):

P=At=U⋅I⋅tt=U⋅I. Таким образом:

Мощность электрического тока равна произведению напряжения на силу тока: P=U⋅I.

Из этой формулы можно определить и другие физические величины.
Для удобства можно использовать приведённые ниже рисунки.

 

 

За единицу мощности принят ватт: \(1\) Вт = \(1\) Дж/с.

 

Из формулы P=U⋅I следует, что


\(1\) ватт = \(1\) вольт ∙ \(1\) ампер, или \(1\) Вт = \(1\) В ∙ А.


 

Обрати внимание!

Используют также единицы мощности, кратные ватту: гектоватт (гВт), киловатт (кВт), мегаватт (МВт).
\(1\) гВт = \(100\) Вт или \(1\) Вт = \(0,01\) гВт;
\(1\) кВт = \(1000\) Вт или \(1\) Вт = \(0,001\) кВт;
\(1\) МВт = \(1 000 000\) Вт или \(1\) Вт = \(0,000001\) МВт.

Пример:

Измерим силу тока в цепи с помощью амперметра, а напряжение на участке — с помощью вольтметра.

 

 

Так как мощность тока прямо пропорциональна напряжению и силе тока, протекающего через лампочку, то перемножим их значения:

 

I=1,2АU=5ВP =U⋅I=5⋅1,2=6Вт.

 

Ваттметры измеряют мощность электрического тока, протекающего через прибор. По своему назначению и техническим характеристикам ваттметры разнообразны.

В зависимости от сферы применения у них различаются пределы измерения.

 

Аналоговый ваттметр

Аналоговый ваттметр

Аналоговый ваттметр

Цифровой ваттметр

 

Подключим к цепи по очереди две лампочки накаливания, сначала одну, затем другую и измерим силу тока в каждой из них. Она будет разной.

 

 

 

Сила тока в лампочке мощностью \(25\) ватт будет составлять \(0,1\) А. Лампочка мощностью \(100\) ватт потребляет ток в четыре раза больше — \(0,4\) А. Напряжение в этом эксперименте неизменно и равно \(220\) В. Легко можно заметить, что лампочка в \(100\) ватт светится гораздо ярче, чем \(25\)-ваттовая лампочка. Это происходит оттого, что её мощность больше. Лампочка, мощность которой в \(4\) раза больше, потребляет в \(4\) раза больше тока. Значит: 

 

Обрати внимание!

Мощность прямо пропорциональна силе тока.

Что произойдёт, если одну и ту же лампочку подсоединить к источникам различного напряжения? В данном случае используется напряжение \(110\) В и \(220\) В.


  

 

Можно заметить, что при большем напряжении лампочка светится ярче, значит, в этом случае её мощность будет больше. Следовательно:

 

Обрати внимание!

Мощность зависит от напряжения.

Рассчитаем мощность лампочки в каждом случае:

 

I=0,2АU=110ВP=U⋅I=110⋅0,2=22ВтI=0,4АU=220ВP=U⋅I=220⋅0,4=88Вт.

 

Можно сделать вывод о том, что при увеличении напряжения в \(2\) раза мощность увеличивается в \(4\) раза.
Не следует путать эту мощность с номинальной мощностью лампы (мощность, на которую рассчитана лампа). Номинальная мощность лампы (а соответственно, ток через нить накала и её расчётное сопротивление) указывается только для номинального напряжения лампы (указано на баллоне, цоколе или упаковке).


 

 

В таблице дана мощность, потребляемая различными приборами и устройствами:

 

Название

Рисунок

Мощность

 Калькулятор

\(0,001\) Вт

 Лампы дневного света

\(15 — 80\) Вт

 Лампы накаливания

\(25 — 5000\) Вт

 Компьютер

\(200 — 450\) Вт

 Электрический чайник

\(650 — 3100\) Вт

 Пылесос

\(1500 — 3000\) Вт

 Стиральная машина

\(2000 — 4000\) Вт

 Трамвай

\(150 000 — 240000\) Вт

физические формулы, использующие мощность и напряжение

При выборе какого-либо электрического оборудования одним из важных параметров, на который обращается внимание, является мощность изделия. Этот параметр неразрывно связан с силой тока и напряжением. Чтобы рассчитать силу тока, напряжение или мощность в электрической цепи, используются несложные формулы. Но чтобы осмысленно проводить такие вычисления, желательно понимать физическую природу возникновения этих величин.

Физическое понятие величин

Любая электрическая цепь характеризуется рядом параметров. Наиболее важными из них являются сила тока, напряжение, мощность и сопротивление. Эти характеристики связаны между собой и зависят друг от друга. Явление, объединяющее их, называется электричеством.

Это понятие было введено ещё в 1600 году английским физиком Уильямом Гилбертом, изучающим магнитные и электрические явления. Исследуя магнетизм в природе, учёный установил, что некоторые тела при трении начинают обладать силой притяжения по отношению к другим предметам, в частности, к янтарю. Поэтому он и назвал открытое явление ēlectricus, что в переводе с латинского обозначает «янтарный».

Продолжая его исследования, немецкий физик Отто фон Герике в 1663 году изобрёл электрическую машину, которая представляла собой металлический стержень с одетым на него серным шаром. В результате он выяснил, что материалы могут не только притягивать вещества, но и отталкивать. Но только через восемьдесят лет американец Бенджамин Франклин создал теорию электричества, введя такие термины, как отрицательный и положительный заряд.

Дальнейшее развитие электричество получило после опытов Шарля Кулона и открытия им закона взаимодействия зарядов. Заключался он в следующем: сила влияния двух точечных зарядов друг на друга в вакууме прямо пропорциональна их произведению и обратно пропорциональна расстоянию между ними в квадрате. После этого благодаря экспериментам таких учёных, как Джоуль, Ленц, Ом, Ампер, Фарадей, Максвелл были введены понятия ток, напряжение и электромагнетизм.

Так, в 1897 году англичанин Джозеф Томсон установил, что носителями зарядов являются электроны. Ранее, в 1880 году, электротехник из России Дмитрий Лачинов сформулировал необходимые условия для передачи электричества на расстояния.

После этих открытий были выработаны фундаментальные определения электричества. Сегодня под ним понимаются свойства материалов образовывать вокруг себя электрическое поле, оказывающее воздействие на располагающиеся рядом другие заряженные частицы. Заряды условно принято разделять на положительные и отрицательные. При их перемещении возникает магнитное поле, при этом одинакового знака заряды притягиваются, а разного — отталкиваются.

Сила тока

Ток — это упорядоченное движение носителей заряда, происходящее под влиянием электрического поля. В качестве положительно заряженных частиц выступают электроны, а отрицательных — дырки. Математически это явление описывается с помощью формулы I = Q*T, где I — ток проводимости (А), Q — заряд частицы (Кл), T — время ©.

То есть электрическим током называется количество зарядов, прошедших через поперечное сечение вещества. Но эта формулировка верна только для тока постоянной величины, в то время как для изменяемого во времени она будет выглядеть I (T) = dQ/dT.

Плотность движения носителей заряда в материале, то есть количество электричества, проходящего за условно принятое время, называется силой тока. Согласно Международной системе (СИ) его единицей измерения является ампер. Один ампер равен перемещению электрического заряда, равного одному кулону, через поперечное сечение за одну секунду.

Носители заряда могут двигаться как упорядоченно, так и хаотично. При их движении возникает электрическое поле, обозначаемое латинской буквой E. Значение, определяющееся отношением тока к поперечному сечению проводника, называется плотностью тока. За единицу её измерения принимается А/мм2.

По своему виду ток различают на следующие типы:

  1. Переноса. Характеризуется движением зарядов, осуществляемым в свободном пространстве. Этот тип характерен для газоразрядных приборов.
  2. Смещения. Возникает в диэлектриках и определяется упорядоченным перемещением связанных заряженных частиц.
  3. Полный. Определяется суммарным значением тока: проводимости, переноса и смещения.
  4. Постоянный. Это такой вид, который может изменять величину, но не изменяет направление движения, то есть свой знак.
  5. Переменный. Такого вида ток может изменяться как по величине, так и по направлению (знаку).

Переменный вид разделяется по форме и может быть синусоидальным и несинусоидальным. Для расчёта силы тока синусоидальной формы используется формула Is = Ia*sin ωt, где Ia — максимальное значение тока (A), ω — угловая скорость, равная 2πf (Гц).

Физические тела, в которых возможно протекание тока, называют проводниками, а в тех, где возникают препятствия его прохождению — диэлектриками. Промежуточное состояние между ними занимают полупроводники.

Разность потенциалов

Напряжением принято называть физическую величину, характеризующую электрическое поле. Она показывает, какую работу понадобится совершить полю для того, чтобы переместить единичный заряд из одной точки в другую. При этом принимается, что этот перенос не влияет на распределение зарядов в источнике поля. Согласно Международной системе единиц напряжение измеряется в вольтах.

Работа по переносу складывается из двух величин — электрических и сторонних. Если сторонние силы не действуют, то напряжение на участке цепи равно разности потенциалов и вычисляется по формуле U = φ1-φ2. При этом потенциал определяется отношением напряжённости электрического поля к заряду. Для его расчёта используют формулу φ = W/q.

Другими словами, это характеристика поля в определённой точке, не зависящей от величины заряда, находящегося в нём. То есть напряжение в общем случае определяется работой электростатического поля, возникающего при движении заряда вдоль его силовых линий. Математически его можно рассчитать по формуле U = A/q, где А — совершаемая работа по перемещению (Дж), q — энергия заряда (Кл).

Применительно к сети переменного тока для напряжения используются следующие понятия:

  1. Мгновенное. Это значение физической величины, измеренное в конкретный момент времени: U = U (t). Для синусоидального сигнала мгновенное напряжение находится с помощью выражения U (t) = Ua sin (ὤt + φ).
  2. Амплитудное. Характеризуется наибольшей величиной мгновенного значения без учёта знака: Ua = max (U (t)).
  3. Среднее. Определяется за полный период сигнала по формуле Us = 1/T ʃ U (t)*dt. Для синусоидальной формы это значение равно нулю.

Проводя расчёт напряжения, редко используется понятие электрического потенциала. Связано это с тем, что условно принято за одну из точек потенциала принимать землю.

Это значение берётся равным нулю, а все остальные потенциалы считаются относительно неё. Говоря, что напряжение в определённой точке составляет 300 вольт, имеется в виду разность потенциалов между этой точкой и землёй, равная этому значению.

Электрическая мощность

Электрическая мощность характеризует скорость передачи электрической энергии или её преобразование. Единицей её измерения является ватт. Для того чтобы посчитать мощность на определённом участке цепи, необходимо перемножить значение напряжения и силы тока на этом участке. Исходя из определения электрического напряжения, можно сказать, что заряд при движении совершает работу, численно равную ей на участке цепи. Если же умножить работу на количество зарядов, то можно найти общее значение работы, которую совершили заряды на этом участке.

Исходя из физического определения, что мощность — это работа за единицу времени, получается выражение P = A/Δt, где A — работа, совершаемая зарядом при перемещении от начальной точки к конечной (Дж), Δt — время, затраченное на полное перемещение заряда ©.

Для всех зарядов в цепи мощность можно найти благодаря формуле P = (U/ Δt) * Q, где Q — общее число зарядов.

Так как ток представляет собой заряд, протекающий в единицу времени (I = Q/ Δt), то получается, что мощность равна произведению тока на напряжение, то есть P = U*I (Вт).

В цепи с постоянным током его сила и напряжение всегда имеют постоянное значение в определённой точке, поэтому для любого момента времени мощность можно вычислить по формуле P = I*U = I2*R = U2/R, где R — сопротивление прохождению тока в электрической цепи (Ом). Если же в этой сети находится источник электродвижущей силы, то мощность находится как P = I*E+ I2*r, где Е — электродвижущая сила или ЭДС (В), r — внутреннее сопротивление источника ЭДС (Ом).

Для цепи, в которой её параметры изменяются по какому-то циклу, мощность в определённой точке интегрируется по времени. При этом существуют следующие виды мощности:

  1. Активная. Для её нахождения используется расчёт, учитывающий угол сдвига фаз φ. Находится согласно формуле P = U*I*cos φ.
  2. Реактивная. Характеризуется нагрузками, создаваемыми электрическими устройствами в виде колебаний энергии электромагнитного поля. Её вычисление осуществляется по формуле P = U*I*sin φ.
  3. Полная. Определяется произведением действующих значений тока и напряжения, связана с другими видами мощности выражением S= √(P 2 +Q 2).

Закон Ома для цепи

Проводя расчёты мощности по напряжению и току на практике, часто используют закон Ома. Он устанавливает связь между током, сопротивлением и напряжением. Этот закон был открыт путём проведения Симоном Омом ряда экспериментов и сформулирован им в 1826 году. Он выяснил, что величина тока на участке цепи прямо пропорциональна разности потенциалов и обратно пропорциональна сопротивлению этого участка.

Закон Ома можно записать в следующем виде: I = U/R, где I — значение силы тока (А), U — разность потенциалов (В), R — сопротивление цепи прохождению тока (Ом).

Для полной же цепи эту формулу можно записать так: I = E/(R+ r0), где E — ЭДС источника питания (В), r0 — внутреннее сопротивление источника напряжения (Ом).

Таким образом, для участка цепи будет справедливо выражение P = U2/R = I2R, а для полной цепи — P = (E/(R+ R0))2*R. Именно эти две формулы и используются чаще всего для расчётов электрических сетей или мощности необходимого оборудования.

Различные компоненты электрической сети в определённый момент времени потребляют разную величину тока. Поэтому очень важно правильно рассчитать, какое количество энергии подводится в тот или иной момент в определённое место цепи, чтобы не допустить перегрузок на линии и возникновения аварийных ситуаций.

Этим и занимаются разработчики схем, упрощая их до состояния, когда можно рассчитать необходимую мощность, используя закон Ома.

Практический расчёт

Например, пусть понадобится узнать, на какой ток необходимо приобрести устанавливаемый на участок цепи автоматический выключатель. При этом известно, что в линию, на которой он будет установлен, одновременно будут включаться холодильник с максимальной мощностью потребления энергии один киловатт, бойлер (два киловатта) и люстра, потребляющая 90 ватт. В месте установки используется однофазная сеть, рассчитанная на рабочее напряжение 220 вольт.

На первом этапе расчёта понадобится суммировать всю мощность подключаемых к линии электроприборов. Так, P общ. = 1000 + 2000 + 90 +220 = 3310 Вт. Используя формулу P = I*U, находится необходимое значение тока: I = P/U = 3310/220 = 15,04 А.

Из стандартного ряда выключателей наиболее близкое значение имеет автомат на 16 А. Поскольку необходимо покупать устройство защиты с небольшим запасом, то для рассматриваемого примера подойдёт выключатель, рассчитанный на 20 ампер.

Благодаря таким вычислениям можно рассчитать любой параметр электрической цепи, но это при учёте достаточного количества вводных данных.

Основные электротехнические формулы. Мощность. Сопротивление. Ток. Напряжение. Закон Ома.

Электрическое напряжение:

  • U = R* I — Закон Ома для участка цепи
  • U = P / I
  • U = (P*R)1/2

Электрическая мощность:

  • P= U* I
  • P= R* I2
  • P = U 2/ R

Электрический ток:

  • I = U / R
  • I = P/ E
  • I = (P / R)1/2

Электрическое сопротивление:

  • R = U / I
  • R = U 2/ P
  • R = P / I2

НЕ ЗАБЫВАЕМ: Законы Кирхгофа они же Правила Кирхгофа для тока и напряжения.

Цепь переменного синусоидального тока c частотой ω.

Применимость формул: пренебрегаем зависимостью сопротивлений от силы тока и частоты.

Напомним, что любой сигнал, может быть с любой точностью разложен в ряд Фурье, т.е. в предположении, что параметры сети
частотнонезависимы — данная формулировка применима ко всем гармоникам любого сигнала.

Закон Ома для цепей переменного тока:

  • U = U0eiωt  напряжение или разность потенциалов,
  • I  сила тока,
  • Z = Reiφ  комплексное сопротивление (импеданс)
  • R = (Ra2+Rr2)1/2  полное сопротивление,
  • Rr = ωL — 1/ωC  реактивное сопротивление (разность индуктивного и емкостного),
  • Rа  активное (омическое) сопротивление, не зависящее от частоты,
  • φ = arctg Rr/Ra — сдвиг фаз между напряжением и током.

Напряжение, ток, сопротивление и закон Ома

Добавлено в избранное Любимый 116

Основы электроэнергетики

Приступая к изучению мира электричества и электроники, важно начать с понимания основ напряжения, тока и сопротивления. Это три основных строительных блока, необходимых для управления электричеством и его использования. Сначала эти концепции могут быть трудными для понимания, потому что мы не можем их «видеть».Невооруженным глазом нельзя увидеть энергию, текущую по проводу, или напряжение батареи, стоящей на столе. Даже молния в небе, хотя и видимая, на самом деле не является обменом энергии между облаками и землей, а является реакцией в воздухе на энергию, проходящую через него. Чтобы обнаружить эту передачу энергии, мы должны использовать измерительные инструменты, такие как мультиметры, анализаторы спектра и осциллографы, чтобы визуализировать, что происходит с зарядом в системе. Однако не бойтесь, это руководство даст вам общее представление о напряжении, токе и сопротивлении, а также о том, как они соотносятся друг с другом.

Георг Ом

Рассмотрено в этом учебном пособии

  • Как электрический заряд соотносится с напряжением, током и сопротивлением.
  • Что такое напряжение, сила тока и сопротивление.
  • Что такое закон Ома и как его использовать для понимания электричества.
  • Простой эксперимент для демонстрации этих концепций.

Рекомендуемая литература

и nbsp

и nbsp

Электрический заряд

Электричество — это движение электронов.Электроны создают заряд, который мы можем использовать для работы. Ваша лампочка, стереосистема, телефон и т. Д. — все используют движение электронов для выполнения работы. Все они работают, используя один и тот же основной источник энергии: движение электронов.

Три основных принципа этого урока можно объяснить с помощью электронов или, более конкретно, заряда, который они создают:

  • Напряжение — это разница в заряде между двумя точками.
  • Текущий — это скорость начисления.
  • Сопротивление — это способность материала сопротивляться прохождению заряда (тока).

Итак, когда мы говорим об этих значениях, мы на самом деле описываем движение заряда и, следовательно, поведение электронов. Цепь — это замкнутая петля, которая позволяет заряду перемещаться из одного места в другое. Компоненты схемы позволяют нам контролировать этот заряд и использовать его для работы.

Георг Ом был баварским ученым, изучавшим электричество. Ом начинается с описания единицы сопротивления, которая определяется током и напряжением.Итак, начнем с напряжения и продолжим.

Напряжение

Мы определяем напряжение как количество потенциальной энергии между двумя точками цепи. Одна точка заряжена больше, чем другая. Эта разница в заряде между двумя точками называется напряжением. Он измеряется в вольтах, что технически представляет собой разность потенциальной энергии между двумя точками, которая будет передавать один джоуль энергии на один кулон заряда, который проходит через нее (не паникуйте, если это не имеет смысла, все будет объяснено).Единица «вольт» названа в честь итальянского физика Алессандро Вольта, который изобрел то, что считается первой химической батареей. Напряжение представлено в уравнениях и схемах буквой «V».

При описании напряжения, тока и сопротивления часто используется аналогия с резервуаром для воды. В этой аналогии заряд представлен количеством воды , напряжение представлено давлением воды , а ток представлен потоком воды . Итак, для этой аналогии запомните:

  • Вода = Заряд
  • Давление = Напряжение
  • Расход = Текущий

Рассмотрим резервуар для воды на определенной высоте над землей.Внизу этого бака есть шланг.

Давление на конце шланга может представлять напряжение. Вода в баке представляет собой заряд. Чем больше воды в баке, тем выше заряд, тем больше давление измеряется на конце шланга.

Мы можем рассматривать этот резервуар как батарею, место, где мы накапливаем определенное количество энергии, а затем высвобождаем ее. Если мы сливаем из нашего бака определенное количество жидкости, давление, создаваемое на конце шланга, падает. Мы можем думать об этом как об уменьшении напряжения, например, когда фонарик тускнеет по мере разрядки батарей.Также уменьшается количество воды, протекающей через шланг. Меньшее давление означает, что течет меньше воды, что приводит нас к течению.

Текущий

Мы можем представить себе количество воды, протекающей по шлангу из бака, как ток. Чем выше давление, тем выше расход, и наоборот. С водой мы бы измерили объем воды, протекающей через шланг за определенный период времени.18 электронов (1 кулон) в секунду проходят через точку в цепи. Ампер в уравнениях обозначается буквой «I».

Предположим теперь, что у нас есть два резервуара, каждый со шлангом, идущим снизу. В каждом резервуаре одинаковое количество воды, но шланг одного резервуара уже, чем шланг другого.

Мы измеряем одинаковое давление на конце любого шланга, но когда вода начинает течь, расход воды в баке с более узким шлангом будет меньше, чем расход воды в баке с более узким шлангом. более широкий шланг.С точки зрения электричества, ток через более узкий шланг меньше, чем ток через более широкий шланг. Если мы хотим, чтобы поток через оба шланга был одинаковым, мы должны увеличить количество воды (заряд) в резервуаре с помощью более узкого шланга.

Это увеличивает давление (напряжение) на конце более узкого шланга, проталкивая больше воды через резервуар. Это аналогично увеличению напряжения, которое вызывает увеличение тока.

Теперь мы начинаем видеть взаимосвязь между напряжением и током.Но здесь следует учитывать третий фактор: ширину шланга. В этой аналогии ширина шланга — это сопротивление. Это означает, что нам нужно добавить еще один термин в нашу модель:

.
  • Вода = заряд (измеряется в кулонах)
  • Давление = напряжение (измеряется в вольтах)
  • Расход = ток (измеряется в амперах, или, для краткости, «амперах»)
  • Ширина шланга = сопротивление

Сопротивление

Снова рассмотрим наши два резервуара для воды, один с узкой трубой, а другой с широкой трубой.

Само собой разумеется, что мы не можем пропустить через узкую трубу такой же объем, как более широкая, при том же давлении. Это сопротивление. Узкая труба «сопротивляется» потоку воды через нее, даже если вода находится под тем же давлением, что и резервуар с более широкой трубой.

С точки зрения электричества это представлено двумя цепями с одинаковым напряжением и разным сопротивлением. Цепь с более высоким сопротивлением позволит протекать меньшему количеству заряда, то есть в цепи с более высоким сопротивлением будет меньше тока, протекающего через нее.18 электронов. Это значение обычно обозначается на схемах греческой буквой «& ohm;», которая называется омега и произносится как «ом».

Закон Ома

Объединив элементы напряжения, тока и сопротивления, Ом разработал формулу:

Где

  • В = Напряжение в вольтах
  • I = ток в амперах
  • R = Сопротивление в Ом

Это называется законом Ома.Скажем, например, что у нас есть цепь с потенциалом 1 вольт, током 1 ампер и сопротивлением 1 Ом. Используя закон Ома, мы можем сказать:

Допустим, это наш резервуар с широким шлангом. Количество воды в баке определяется как 1 вольт, а «узость» (сопротивление потоку) шланга определяется как 1 ом. Используя закон Ома, это дает нам ток (ток) в 1 ампер.

Используя эту аналогию, давайте теперь посмотрим на резервуар с узким шлангом. Поскольку шланг более узкий, его сопротивление потоку выше.Определим это сопротивление как 2 Ом. Количество воды в резервуаре такое же, как и в другом резервуаре, поэтому, используя закон Ома, наше уравнение для резервуара с узким шлангом составляет

.

а какой ток? Поскольку сопротивление больше, а напряжение такое же, это дает нам значение тока 0,5 А:

Значит, в баке с большим сопротивлением ток меньше. Теперь мы видим, что, зная два значения закона Ома, мы можем решить третье.Продемонстрируем это на эксперименте.

Эксперимент по закону Ома

Для этого эксперимента мы хотим использовать батарею на 9 В для питания светодиода. Светодиоды хрупкие и могут пропускать только определенное количество тока, прежде чем они перегорят. В документации к светодиоду всегда будет «текущий рейтинг». Это максимальное количество тока, которое может пройти через конкретный светодиод, прежде чем он перегорит.

Необходимые материалы

Для проведения экспериментов, перечисленных в конце руководства, вам потребуется:

ПРИМЕЧАНИЕ. Светодиоды — это так называемые «неомические» устройства.Это означает, что уравнение для тока, протекающего через сам светодиод, не так просто, как V = IR. Светодиод вызывает в цепи то, что называется «падением напряжения», тем самым изменяя величину протекающего через нее тока. Однако в этом эксперименте мы просто пытаемся защитить светодиод от перегрузки по току, поэтому мы пренебрегаем токовыми характеристиками светодиода и выбираем значение резистора, используя закон Ома, чтобы быть уверенным, что ток через светодиод безопасно ниже 20 мА.

В этом примере у нас есть батарея на 9 В и красный светодиод с номинальным током 20 мА, или 0.020 ампер. Чтобы быть в безопасности, мы бы предпочли не управлять максимальным током светодиода, а его рекомендуемым током, который указан в его техническом описании как 18 мА или 0,018 ампер. Если просто подключить светодиод напрямую к батарее, значения закона Ома будут выглядеть так:

следовательно:

, а поскольку сопротивления еще нет:

Деление на ноль дает бесконечный ток! Что ж, на практике не бесконечно, но столько тока, сколько может доставить аккумулятор. Поскольку мы НЕ хотим, чтобы через светодиод проходил такой большой ток, нам понадобится резистор.Наша схема должна выглядеть так:

Мы можем использовать закон Ома точно так же, чтобы определить значение резистора, которое даст нам желаемое значение тока:

следовательно:

вставляем наши значения:

решение для сопротивления:

Итак, нам нужно сопротивление резистора около 500 Ом, чтобы ток через светодиод не превышал максимально допустимый.

500 Ом не является обычным значением для стандартных резисторов, поэтому в этом устройстве вместо него используется резистор 560 Ом.Вот как выглядит наше устройство вместе.

Успех! Мы выбрали номинал резистора, который достаточно высок, чтобы ток через светодиод не превышал его максимального номинала, но достаточно низкий, чтобы ток был достаточным, чтобы светодиод оставался красивым и ярким.

Этот пример светодиодного / токоограничивающего резистора является обычным явлением в хобби-электронике. Вам часто придется использовать закон Ома, чтобы изменить величину тока, протекающего по цепи. Другой пример такой реализации — светодиодные платы LilyPad.

При такой настройке вместо того, чтобы выбирать резистор для светодиода, резистор уже встроен в светодиод, поэтому ограничение тока выполняется без необходимости добавлять резистор вручную.

Ограничение тока до или после светодиода?

Чтобы немного усложнить задачу, вы можете разместить токоограничивающий резистор по обе стороны от светодиода, и он будет работать точно так же!

Многие люди, впервые изучающие электронику, борются с идеей, что резистор, ограничивающий ток, может находиться по обе стороны от светодиода, и схема по-прежнему будет работать в обычном режиме.

Представьте себе реку в непрерывной петле, бесконечную, круглую, текущую реку. Если бы мы построили в нем плотину, вся река перестала бы течь, а не только с одной стороны. Теперь представьте, что мы помещаем водяное колесо в реку, которое замедляет течение реки. Неважно, где в круге находится водяное колесо, оно все равно замедлит поток на всей реке .

Это чрезмерное упрощение, поскольку резистор ограничения тока нельзя размещать где-либо в цепи ; он может быть размещен на с любой стороны светодиода для выполнения своей функции.

Чтобы получить более научный ответ, мы обратимся к закону напряжения Кирхгофа. Именно из-за этого закона резистор, ограничивающий ток, может располагаться по обе стороны светодиода и при этом иметь тот же эффект. Для получения дополнительной информации и некоторых практических задач с использованием KVL посетите этот веб-сайт.

Ресурсы и дальнейшее развитие

Теперь вы должны понять концепции напряжения, тока, сопротивления и их взаимосвязь. Поздравляю! Большинство уравнений и законов для анализа цепей можно вывести непосредственно из закона Ома.Зная этот простой закон, вы понимаете концепцию, лежащую в основе анализа любой электрической цепи!

Эти концепции — лишь верхушка айсберга. Если вы хотите продолжить изучение более сложных приложений закона Ома и проектирования электрических цепей, обязательно ознакомьтесь со следующими руководствами.

Напряжение и ток | Основные понятия электричества

Как упоминалось ранее, нам нужно нечто большее, чем просто непрерывный путь (т.е.е., цепь) до того, как возникнет непрерывный поток заряда: нам также нужны средства, чтобы протолкнуть эти носители заряда по цепи. Так же, как мрамор в трубе или вода в трубе, для инициирования потока требуется некоторая сила воздействия. В случае электронов эта сила — это та же сила, которая действует в статическом электричестве: сила, создаваемая дисбалансом электрического заряда. Если мы возьмем примеры воска и шерсти, которые были натерты друг с другом, мы обнаружим, что избыток электронов в воске (отрицательный заряд) и дефицит электронов в шерсти (положительный заряд) создают дисбаланс заряда между ними.Этот дисбаланс проявляется как сила притяжения между двумя объектами:

Если между заряженным воском и шерстью поместить проводящую проволоку, электроны будут проходить через нее, так как некоторые из избыточных электронов в воске устремляются через провод, чтобы вернуться к шерсти, восполняя там недостаток электронов:

Дисбаланс электронов между атомами воска и атомами шерсти создает силу между двумя материалами. Поскольку электроны не могут перетекать от воска к шерсти, все, что может сделать эта сила, — это притягивать два объекта вместе.Однако теперь, когда проводник перекрывает изолирующий зазор, сила заставит электроны течь в однородном направлении через провод, хотя бы на мгновение, пока заряд в этой области не нейтрализуется и сила между воском и шерстью не уменьшится. Электрический заряд, образованный между этими двумя материалами при трении их друг о друга, служит для хранения определенного количества энергии. Эта энергия мало чем отличается от энергии, накопленной в высоком резервуаре с водой, который выкачивается из пруда нижнего уровня:

Влияние силы тяжести на воду в резервуаре создает силу, которая пытается снова опустить воду на более низкий уровень.Если подходящая труба проложена от резервуара обратно к пруду, вода под действием силы тяжести потечет вниз из резервуара по трубе:

Для перекачки этой воды из пруда с низким уровнем в резервуар с высоким уровнем требуется энергия, а движение воды по трубопроводу обратно к исходному уровню представляет собой высвобождение энергии, накопленной от предыдущей откачки. Если вода перекачивается на еще более высокий уровень, для этого потребуется еще больше энергии, таким образом, будет сохранено больше энергии, и больше энергии будет высвобождено, если воде позволить снова течь по трубе обратно вниз:

Электроны мало чем отличаются.Если мы протираем воск и шерсть вместе, мы «выкачиваем» электроны с их нормальных «уровней», создавая условия, при которых существует сила между парафином и шерстью, поскольку электроны стремятся восстановить свои прежние положения (и балансировать внутри своего тела). соответствующие атомы). Сила, притягивающая электроны обратно в их исходное положение вокруг положительных ядер их атомов, аналогична силе гравитации, действующей на воду в резервуаре, пытаясь вернуть ее к прежнему уровню. Подобно тому, как перекачка воды на более высокий уровень приводит к накоплению энергии, «перекачка» электронов для создания дисбаланса электрического заряда приводит к накоплению определенного количества энергии в этом дисбалансе.И точно так же, как предоставление возможности воде стекать обратно с высоты резервуара приводит к высвобождению этой накопленной энергии, предоставление возможности электронам течь обратно к их первоначальным «уровням» приводит к высвобождению накопленной энергии. Когда носители заряда находятся в этом статическом состоянии (точно так же, как вода, неподвижная, высоко в резервуаре), энергия, хранящаяся там, называется потенциальной энергией , потому что у нее есть возможность (потенциал) высвобождения, которая не была полностью реализована. все же.

Понятие о напряжении

Когда носители заряда находятся в статическом состоянии (как вода, неподвижная, высоко в резервуаре), энергия, хранящаяся там, называется потенциальной энергией, потому что у нее есть возможность (потенциал) высвобождения, которая еще не полностью реализована. . Когда вы терзаете обувь с резиновой подошвой о тканевый ковер в сухой день, вы создаете дисбаланс электрического заряда между вами и ковром. При царапании ногами накапливается энергия в виде дисбаланса зарядов, вытесняемых из их первоначальных мест.Этот заряд (статическое электричество) является стационарным, и вы вообще не заметите, что энергия накапливается. Однако, как только вы положите руку на металлическую дверную ручку (с большой подвижностью электронов для нейтрализации вашего электрического заряда), эта накопленная энергия будет высвобождена в виде внезапного потока заряда через вашу руку, и вы будете воспринимать ее как поражение электрическим током! Эта потенциальная энергия, хранящаяся в виде дисбаланса электрического заряда и способная провоцировать протекание носителей заряда через проводник, может быть выражена термином, называемым напряжением, которое технически является мерой потенциальной энергии на единицу заряда или чего-то, что физик мог бы называют удельной потенциальной энергией.

Определение напряжения

Определяемое в контексте статического электричества, напряжение — это мера работы, необходимой для перемещения единичного заряда из одного места в другое, против силы, которая пытается сохранить баланс электрических зарядов. В контексте источников электроэнергии напряжение — это количество доступной потенциальной энергии (работа, которую необходимо выполнить) на единицу заряда для перемещения зарядов через проводник, поскольку напряжение — это выражение потенциальной энергии, представляющее возможность или потенциал для выделения энергии когда заряд перемещается с одного «уровня» на другой, он всегда находится между двумя точками.Рассмотрим аналогию с водохранилищем:

.

Из-за разницы в высоте падения существует вероятность того, что гораздо больше энергии будет выпущено из резервуара через трубопровод в точку 2, чем в точку 1. Принцип интуитивно понятен при падении камня: что приводит к более сильный удар, камень упал с высоты одного фута или тот же камень упал с высоты одной мили? Очевидно, что падение с большей высоты приводит к высвобождению большей энергии (более сильному удару).Мы не можем оценить количество накопленной энергии в водохранилище, просто измерив объем воды, точно так же, как мы можем предсказать серьезность удара падающей породы, просто зная вес породы: в обоих случаях мы также должны учитывать, как далекие эти массы упадут со своей начальной высоты. Количество энергии, высвобождаемой при падении массы, зависит от расстояния между его начальной и конечной точками. Точно так же потенциальная энергия, доступная для перемещения носителей заряда из одной точки в другую, зависит от этих двух точек.Следовательно, напряжение всегда выражается как величина между двумя точками. Интересно, что аналогия с массой, потенциально «падающей» с одной высоты на другую, является настолько удачной моделью, что напряжение между двумя точками иногда называют падением напряжения .

Генерирующее напряжение

Напряжение можно генерировать другими способами, кроме трения материалов определенных типов друг о друга. Химические реакции, лучистая энергия и влияние магнетизма на проводники — вот несколько способов создания напряжения.Соответствующими примерами этих трех источников напряжения являются батареи, солнечные элементы и генераторы (например, «генератор переменного тока» под капотом вашего автомобиля). На данный момент мы не будем вдаваться в подробности того, как работает каждый из этих источников напряжения — более важно то, что мы понимаем, как источники напряжения могут применяться для создания потока заряда в электрической цепи. Давайте возьмем символ химической батареи и шаг за шагом построим схему:

Как работают источники напряжения?

Любой источник напряжения, включая аккумуляторные батареи, имеет две точки электрического контакта.В этом случае у нас есть точка 1 и точка 2 на приведенной выше диаграмме. Горизонтальные линии разной длины указывают на то, что это батарея, и дополнительно указывают направление, в котором напряжение этой батареи будет пытаться протолкнуть носители заряда по цепи. Тот факт, что горизонтальные линии в символе батареи кажутся разделенными (и, таким образом, не могут служить в качестве пути для потока заряда), не вызывает беспокойства: в реальной жизни эти горизонтальные линии представляют собой металлические пластины, погруженные в жидкий или полутвердый материал. который не только проводит заряды, но и генерирует напряжение, чтобы подтолкнуть их, взаимодействуя с пластинами.Обратите внимание на маленькие значки «+» и «-» непосредственно слева от символа батареи. Отрицательный (-) конец батареи всегда является концом с самым коротким тире, а положительный (+) конец батареи всегда является концом с самым длинным тире. Положительный конец батареи — это конец, который пытается вытолкнуть из нее носители заряда (помните, что по традиции мы думаем, что носители заряда заряжены положительно, хотя электроны заряжены отрицательно). Точно так же отрицательный конец — это конец, который пытается привлечь носители заряда.Когда «+» и «-» концы батареи ни к чему не подключены, между этими двумя точками будет напряжение, но не будет потока заряда через батарею, потому что нет непрерывного пути, по которому могут перемещаться носители заряда.

Тот же принцип справедлив и для аналогии с резервуаром для воды и насосом: без возвратной трубы обратно в пруд накопленная энергия в резервуаре не может быть выпущена в виде потока воды. Когда резервуар полностью заполнен, поток не может возникнуть, независимо от того, какое давление может создать насос.Должен существовать полный путь (контур), по которому вода течет из пруда в резервуар и обратно в пруд для обеспечения непрерывного потока. Мы можем обеспечить такой путь для батареи, соединив кусок провода от одного конца батареи к другому. Формируя цепь с петлей из проволоки, мы инициируем непрерывный поток заряда по часовой стрелке:

Понимание концепции электрического тока

Пока батарея продолжает вырабатывать напряжение и непрерывность электрического пути не нарушена, носители заряда будут продолжать течь в цепи.Следуя метафоре воды, движущейся по трубе, этот непрерывный, равномерный поток заряда через цепь называется током . Пока источник напряжения продолжает «толкать» в одном направлении, носители заряда будут продолжать двигаться в том же направлении в цепи. Этот однонаправленный поток тока называется , постоянный ток, , или постоянный ток. Во втором томе этой серии книг исследуются электрические цепи, в которых направление тока переключается взад и вперед: переменного тока, или переменного тока.Но пока мы просто займемся цепями постоянного тока. Поскольку электрический ток состоит из отдельных носителей заряда, протекающих в унисон через проводник, перемещаясь и толкая носители заряда впереди, точно так же, как шарики через трубу или вода через трубу, величина потока в одной цепи будет одинаковой. в любой точке. Если бы мы отслеживали поперечное сечение провода в одной цепи, считая протекающие носители заряда, мы бы заметили точно такое же количество в единицу времени, что и в любой другой части цепи, независимо от длины проводника или проводника. диаметр.Если мы нарушим непрерывность цепи в любой точке , электрический ток прекратится во всей петле, и полное напряжение, произведенное батареей, будет проявляться через разрыв между концами проводов, которые раньше были соединены:

Что такое полярность падения напряжения?

Обратите внимание на знаки «+» и «-», нарисованные на концах разрыва цепи, и то, как они соответствуют знакам «+» и «-» рядом с выводами аккумулятора. Эти маркеры указывают направление, в котором напряжение пытается подтолкнуть ток, это направление потенциала, обычно называемое , полярность .Помните, что напряжение всегда относительно между двумя точками. По этой причине полярность падения напряжения также является относительной между двумя точками: будет ли точка в цепи помечена знаком «+» или «-», зависит от другой точки, к которой она относится. Взгляните на следующую схему, где каждый угол петли отмечен номером для справки:

При нарушении целостности цепи между точками 2 и 3 полярность падения напряжения между точками 2 и 3 будет «+» для точки 2 и «-» для точки 3.Полярность батареи (1 «+» и 4 «-») пытается протолкнуть ток через петлю по часовой стрелке от 1 до 2, от 3 до 4 и снова обратно к 1. Теперь давайте посмотрим, что произойдет, если мы снова соединим точки 2 и 3 вместе, но сделаем разрыв в цепи между точками 3 и 4:

При разрыве между 3 и 4 полярность падения напряжения между этими двумя точками будет «-» для 4 и «+» для 3. Обратите особое внимание на тот факт, что «знак» точки 3 противоположен знаку в Первый пример, где разрыв был между точками 2 и 3 (где точка 3 была помечена «-»).Мы не можем сказать, что точка 3 в этой цепи всегда будет либо «+», либо «-», потому что полярность, как и само напряжение, не зависит от одной точки, а всегда относительна между двумя точками!

ОБЗОР:

  • Носители заряда могут двигаться через проводник с помощью той же силы, которая проявляется в статическом электричестве.
  • Напряжение — это мера удельной потенциальной энергии (потенциальной энергии на единицу заряда) между двумя точками.С точки зрения непрофессионала, это мера «толчка», позволяющая мотивировать обвинение.
  • Напряжение, как выражение потенциальной энергии, всегда является относительным между двумя местоположениями или точками. Иногда это называют «падением напряжения».
  • Когда источник напряжения подключен к цепи, напряжение вызывает равномерный поток носителей заряда через эту цепь, называемый током .
  • В одиночной (однопетлевой) схеме величина тока в любой точке такая же, как и величина тока в любой другой точке.
  • При разрыве цепи, содержащей источник напряжения, полное напряжение этого источника появится в точках разрыва.
  • +/- ориентация падения напряжения называется полярностью . Это также относительное значение между двумя точками.

СВЯЗАННЫЕ РАБОЧИЕ ЛИСТЫ:

Электроэнергия и энергия | Безграничная физика

Энергопотребление

Используемая энергия — это временной интеграл от электрической мощности.

Цели обучения

Сформулируйте взаимосвязь между использованием энергии и электрической мощностью

Основные выводы

Ключевые моменты
  • Напомним, что мощность — это скорость выполнения работы или скорость, с которой энергия потребляется или производится.По току и напряжению P = IV.
  • Используемая энергия — это количество заряда q, прошедшего через напряжение V за интервал времени t. Он равен интегралу мощности во времени.
  • Распространенной единицей, используемой для описания использования энергии, является киловатт-час, энергия 1000 Вт, действующая в течение одного часа.
Ключевые термины
  • киловатт-час : единица электрической энергии, равная мощности одного киловатта, действующего в течение одного часа; равняется 3,6 мегаджоулей.Обозначение: кВтч.

Во многих случаях необходимо рассчитать потребление энергии электрическим устройством или набором устройств, например, в доме. Например, мы (или энергокомпания) можем захотеть рассчитать сумму задолженности за потребленную электроэнергию. В другом случае нам может потребоваться определить энергию, необходимую для питания компонента или устройства в течение заданного периода времени. Последнее различие имеет решающее значение — энергия, используемая схемой или компонентом, равна интегралу по времени от электрической мощности .

Мощность

Напомним, что мощность — это скорость выполнения работы или скорость, с которой энергия потребляется или производится, и измеряется в ваттах (Вт). Электрическая мощность в ваттах, вырабатываемая электрическим током I, состоящим из заряда Q кулонов каждые t секунд, проходящего через разность электрических потенциалов (напряжений) V, равна [латекс] \ text {P} = \ frac {\ text {QV} } {\ text {t}} = \ text {IV} [/ latex], где Q — электрический заряд в кулонах, t — время в секундах, I — электрический ток в амперах, а V — электрический потенциал или напряжение в вольтах.2} {\ text {R}} [/ latex], где R — электрическое сопротивление. Власть не обязательно постоянна; он может меняться со временем. Тогда общее выражение для электроэнергии

.

[латекс] \ text {P} (\ text {t}) = \ text {I} (\ text {t}) \ text {V} (\ text {t}) [/ latex]

, где ток I и напряжение V могут изменяться во времени.

Энергия

В любом заданном временном интервале потребляемая (или предоставляемая, в зависимости от вашей точки зрения) энергия определяется выражением [latex] \ text {PE} = \ text {qV} [/ latex], где E — электрическая энергия, V — напряжение, а q — количество заряда, перемещенного за рассматриваемый интервал времени.Мы можем связать общую потребляемую энергию с мощностью, интегрируя по времени: Положительная энергия соответствует потребляемой энергии, а отрицательная энергия соответствует производству энергии. Обратите внимание, что элемент схемы, имеющий как положительный, так и отрицательный профиль мощности в течение некоторого промежутка времени, может потреблять или производить энергию в соответствии со знаком интеграла мощности. Если мощность постоянна в течение временного интервала, то энергию можно просто выразить как:

[латекс] \ text {E} = \ text {Pt} [/ latex].

Единицы потребления энергии

Мы, конечно, хорошо знакомы с единицей измерения энергии в системе СИ — джоуль. Однако, как правило, в счетах за электроэнергию домохозяйства указывается потребление энергии в киловатт-часах (кВтч). Кроме того, это устройство часто встречается в других местах, когда рассматривается использование энергии энергопотребляющими устройствами, структурами или юрисдикциями. Мы можем проанализировать преобразование киловатт-часов в джоули следующим образом: 1 Вт = 1 Дж / с, киловатт равен 1000 Вт, а один час равен 3600 секундам, поэтому 1 кВт-ч равен (1000 Дж / с) (3600 с). = 3 600 000 джоулей.Это масштаб домашнего использования энергии в США, который составляет порядка сотен киловатт-часов в месяц.

Снижение потребления энергии

Потребляемая электрическая энергия (E) может быть уменьшена либо за счет сокращения времени использования, либо за счет снижения энергопотребления этого прибора или приспособления. Это не только снизит стоимость, но и снизит воздействие на окружающую среду. Улучшение освещения — один из самых быстрых способов снизить потребление электроэнергии в доме или на работе.Около 20% энергии в доме расходуется на освещение, в то время как в коммерческих учреждениях эта цифра приближается к 40%. Флуоресцентные лампы примерно в четыре раза эффективнее ламп накаливания — это верно как для длинных ламп, так и для компактных люминесцентных ламп (КЛЛ). Таким образом, лампу накаливания мощностью 60 Вт можно заменить КЛЛ мощностью 15 Вт, которая имеет такую ​​же яркость и цвет. КЛЛ имеют изогнутую трубку внутри шара или спиралевидную трубку, соединенную со стандартным резьбовым основанием, подходящим для стандартных розеток лампы накаливания.(Первоначальные проблемы с цветом, мерцанием, формой и высокими начальными затратами на КЛЛ были решены в последние годы.) Теплопередача от этих КЛЛ меньше, и они служат до 10 раз дольше.

Компактный люминесцентный светильник (КЛЛ) : КЛЛ намного более эффективны, чем лампы накаливания, и поэтому потребляют гораздо меньше энергии для получения яркого света.

Электроэнергия и энергия | Физика

Цели обучения

К концу этого раздела вы сможете:

  • Рассчитайте мощность, рассеиваемую резистором, и мощность, подаваемую источником питания.
  • Рассчитайте стоимость электроэнергии при различных обстоятельствах.

Мощность в электрических цепях

У многих людей власть ассоциируется с электричеством. Зная, что мощность — это коэффициент использования или преобразования энергии, каково выражение для электроэнергии ? На ум могут прийти линии электропередач. Мы также думаем о лампочках с точки зрения их номинальной мощности в ваттах. Сравним лампочку на 25 Вт с лампой на 60 Вт.(См. Рис. 1 (а).) Поскольку оба работают от одного и того же напряжения, лампа мощностью 60 Вт должна потреблять больше тока, чтобы иметь большую номинальную мощность. Таким образом, сопротивление лампы на 60 Вт должно быть ниже, чем у лампы на 25 Вт. Если мы увеличиваем напряжение, мы также увеличиваем мощность. Например, когда лампочка мощностью 25 Вт, рассчитанная на работу от 120 В, подключена к 240 В, она на короткое время очень ярко светится, а затем перегорает. Как именно напряжение, ток и сопротивление связаны с электроэнергией?

Рис. 1. (a) Какая из этих лампочек, лампа мощностью 25 Вт (вверху слева) или лампа мощностью 60 Вт (вверху справа), имеет более высокое сопротивление? Что потребляет больше тока? Что потребляет больше всего энергии? Можно ли по цвету сказать, что нить накаливания мощностью 25 Вт круче? Является ли более яркая лампочка другого цвета, и если да, то почему? (кредиты: Дикбаух, Wikimedia Commons; Грег Вестфолл, Flickr) (б) Этот компактный люминесцентный светильник (КЛЛ) излучает такую ​​же интенсивность света, как и лампа мощностью 60 Вт, но с входной мощностью от 1/4 до 1/10.(кредит: dbgg1979, Flickr)

Электрическая энергия зависит как от напряжения, так и от перемещаемого заряда. Проще всего это выражается как PE = qV , где q — это перемещенный заряд, а V — это напряжение (или, точнее, разность потенциалов, через которую проходит заряд). Мощность — это скорость перемещения энергии, поэтому электрическая мощность равна

.

[латекс] P = \ frac {PE} {t} = \ frac {qV} {t} \\ [/ latex].

Учитывая, что ток равен I = q / t (обратите внимание, что Δ t = t здесь), выражение для мощности принимает вид

P = IV

Электрическая мощность ( P ) — это просто произведение тока на напряжение.Мощность имеет знакомые единицы ватт. Поскольку единицей СИ для потенциальной энергии (PE) является джоуль, мощность выражается в джоулях в секунду или ваттах. Таким образом, 1 A ⋅V = 1 Вт. Например, в автомобилях часто есть одна или несколько дополнительных розеток, с помощью которых можно заряжать сотовый телефон или другие электронные устройства. {2} R \\ [/ latex].

Обратите внимание, что первое уравнение всегда верно, тогда как два других можно использовать только для резисторов. В простой схеме с одним источником напряжения и одним резистором мощность, подаваемая источником напряжения, и мощность, рассеиваемая резистором, идентичны. (В более сложных схемах P может быть мощностью, рассеиваемой одним устройством, а не полной мощностью в цепи.) Из трех различных выражений для электрической мощности можно получить различное понимание. Например, P = В 2 / R означает, что чем ниже сопротивление, подключенное к данному источнику напряжения, тем больше передаваемая мощность.Кроме того, поскольку напряжение возведено в квадрат в P = В 2 / R , эффект от приложения более высокого напряжения, возможно, больше, чем ожидалось. Таким образом, когда напряжение увеличивается вдвое до лампочки мощностью 25 Вт, ее мощность увеличивается почти в четыре раза и составляет примерно 100 Вт, что приводит к ее перегоранию. Если бы сопротивление лампы оставалось постоянным, ее мощность была бы ровно 100 Вт, но при более высокой температуре ее сопротивление также будет выше.

Пример 1. Расчет рассеиваемой мощности и тока: горячая и холодная мощность

(а) Рассмотрим примеры, приведенные в Законе Ома: сопротивление и простые цепи и сопротивление и удельное сопротивление.Затем найдите мощность, рассеиваемую автомобильной фарой в этих примерах, как в горячую, так и в холодную погоду. б) Какой ток он потребляет в холодном состоянии?

Стратегия для (а)

Для горячей фары нам известны напряжение и ток, поэтому мы можем использовать P = IV , чтобы найти мощность. Для холодной фары нам известны напряжение и сопротивление, поэтому мы можем использовать P = В 2 / R , чтобы найти мощность.

Решение для (a)

Вводя известные значения тока и напряжения для горячей фары, получаем

P = IV = (2.{2}} {0,350 \ text {} \ Omega} = 411 \ text {W} \\ [/ latex].

Обсуждение для (а)

30 Вт, рассеиваемые горячей фарой, являются типичными. Но 411 Вт в холодную погоду на удивление выше. Начальная мощность быстро уменьшается по мере увеличения температуры лампы и увеличения ее сопротивления.

Стратегия и решение для (b)

Ток при холодной лампочке можно найти несколькими способами. Переставляем одно из уравнений мощности, P = I 2 R , и вводим известные значения, получая

[латекс] I = \ sqrt {\ frac {P} {R}} = \ sqrt {\ frac {411 \ text {W}} {{0.350} \ text {} \ Omega}} = 34,3 \ text {A} \\ [/ latex].

Обсуждение для (б)

Холодный ток значительно выше, чем установившееся значение 2,50 А, но ток будет быстро снижаться до этого значения по мере увеличения температуры лампы. Большинство предохранителей и автоматических выключателей (используемых для ограничения тока в цепи) рассчитаны на кратковременную выдержку очень высоких токов при включении устройства. В некоторых случаях, например, с электродвигателями, ток остается высоким в течение нескольких секунд, что требует использования специальных плавких предохранителей с замедленным срабатыванием.

Чем больше электроприборов вы используете и чем дольше они остаются включенными, тем выше ваш счет за электроэнергию. Этот знакомый факт основан на соотношении энергии и мощности. Вы платите за использованную энергию. Так как P = E / t , мы видим, что

E = Pt

— это энергия, используемая устройством, использующим мощность P в течение временного интервала t . Например, чем больше горело лампочек, тем больше использовалось P ; чем дольше они работают, тем больше т .Единицей измерения энергии в счетах за электроэнергию является киловатт-час (кВт ч), что соответствует соотношению E = Pt . Стоимость эксплуатации электроприборов легко оценить, если у вас есть некоторое представление об их потребляемой мощности в ваттах или киловаттах, времени их работы в часах и стоимости киловатт-часа для вашей электросети. Киловатт-часы, как и все другие специализированные единицы энергии, такие как пищевые калории, можно преобразовать в джоули. Вы можете доказать себе, что 1 кВт ⋅ ч = 3.6 × 10 6 Дж.

Потребляемая электрическая энергия ( E ) может быть уменьшена либо за счет сокращения времени использования, либо за счет снижения энергопотребления этого прибора или приспособления. Это не только снизит стоимость, но и снизит воздействие на окружающую среду. Улучшение освещения — один из самых быстрых способов снизить потребление электроэнергии в доме или на работе. Около 20% энергии в доме расходуется на освещение, в то время как в коммерческих учреждениях эта цифра приближается к 40%.Флуоресцентные лампы примерно в четыре раза эффективнее ламп накаливания — это верно как для длинных ламп, так и для компактных люминесцентных ламп (КЛЛ). (См. Рис. 1 (b).) Таким образом, лампу накаливания мощностью 60 Вт можно заменить на КЛЛ мощностью 15 Вт, которая имеет такую ​​же яркость и цвет. КЛЛ имеют изогнутую трубку внутри шара или спиралевидную трубку, соединенную со стандартным резьбовым основанием, подходящим для стандартных розеток лампы накаливания. (В последние годы были решены оригинальные проблемы с цветом, мерцанием, формой и высокими начальными затратами на КЛЛ.) Теплопередача от этих КЛЛ меньше, и они служат до 10 раз дольше. В следующем примере рассматривается важность инвестиций в такие лампы. Новые белые светодиодные фонари (представляющие собой группу небольших светодиодных лампочек) еще более эффективны (в два раза больше, чем у КЛЛ) и служат в 5 раз дольше, чем КЛЛ. Однако их стоимость по-прежнему высока.

Установление соединений: энергия, мощность и время

Отношение E = Pt может оказаться полезным во многих различных контекстах.Энергия, которую ваше тело использует во время упражнений, зависит, например, от уровня мощности и продолжительности вашей активности. Степень нагрева источника питания зависит от уровня мощности и времени ее применения. Даже доза облучения рентгеновского изображения зависит от мощности и времени воздействия.

Пример 2. Расчет рентабельности компактных люминесцентных ламп (КЛЛ)

Если стоимость электроэнергии в вашем районе составляет 12 центов за кВтч, какова общая стоимость (капитальные плюс эксплуатация) использования лампы накаливания мощностью 60 Вт в течение 1000 часов (срок службы этой лампы), если стоимость лампы составляет 25 центов? (б) Если мы заменим эту лампочку компактной люминесцентной лампой, которая обеспечивает такой же световой поток, но составляет четверть мощности и стоит 1 доллар.50, но длится в 10 раз дольше (10 000 часов), какова будет общая стоимость?

Стратегия

Чтобы найти эксплуатационные расходы, мы сначала находим использованную энергию в киловатт-часах, а затем умножаем ее на стоимость киловатт-часа.

Решение для (a)

Энергия, используемая в киловатт-часах, определяется путем ввода мощности и времени в выражение для энергии:

E = Pt = (60 Вт) (1000 ч) = 60,000 Вт ⋅ ч

В киловатт-часах это

E = 60.0 кВт ⋅ ч.

Сейчас стоимость электроэнергии

Стоимость

= (60,0 кВт ч) (0,12 долл. США / кВт час) = 7,20 долл. США.

Общая стоимость составит 7,20 доллара за 1000 часов (около полугода при 5 часах в день).

Решение для (b)

Поскольку CFL использует только 15 Вт, а не 60 Вт, стоимость электроэнергии составит 7,20 доллара США / 4 = 1,80 доллара США. КЛЛ прослужит в 10 раз дольше, чем лампа накаливания, так что инвестиционные затраты составят 1/10 стоимости лампы за этот период использования, или 0.1 (1,50 доллара США) = 0,15 доллара США. Таким образом, общая стоимость 1000 часов составит 1,95 доллара США.

Обсуждение

Следовательно, использование КЛЛ намного дешевле, даже если начальные вложения выше. Повышенная стоимость рабочей силы, которую бизнес должен включать в себя для более частой замены ламп накаливания, здесь не учитывается.

Выполнение подключений: эксперимент на вынос — инвентаризация использования электроэнергии

1) Составьте список номинальной мощности для ряда приборов в вашем доме или комнате.Объясните, почему что-то вроде тостера имеет более высокий рейтинг, чем цифровые часы. Оцените энергию, потребляемую этими приборами в среднем за день (оценивая время их использования). Некоторые приборы могут указывать только рабочий ток. Если бытовое напряжение составляет 120 В, тогда используйте P = IV . 2) Проверьте общую мощность, используемую в туалетах на этаже или в здании вашей школы. (Возможно, вам придется предположить, что используемые длинные люминесцентные лампы рассчитаны на 32 Вт.) Предположим, что здание было закрыто все выходные, и что эти огни были оставлены включенными с 6 часов вечера.{2} R \\ [/ латекс].

  • Энергия, используемая устройством с мощностью P за время t , составляет E = Pt .

Концептуальные вопросы

1. Почему лампы накаливания тускнеют в конце жизни, особенно незадолго до того, как их нити оборвутся?

Мощность, рассеиваемая на резисторе, равна P = V 2 / R , что означает, что мощность уменьшается при увеличении сопротивления. Тем не менее, эта мощность также определяется соотношением P = I 2 R , что означает, что мощность увеличивается при увеличении сопротивления.Объясните, почему здесь нет противоречия.

Задачи и упражнения

1. Какова мощность разряда молнии 1,00 × 10 2 МВ при токе 2,00 × 10 4 A ?

2. Какая мощность подается на стартер большого грузовика, который потребляет ток 250 А от аккумуляторной батареи 24,0 В?

3. Заряд в 4,00 Кл проходит через солнечные элементы карманного калькулятора за 4,00 часа. Какова выходная мощность, если выходное напряжение вычислителя равно 3.00 В? (См. Рисунок 2.)

Рис. 2. Полоса солнечных элементов прямо над клавишами этого калькулятора преобразует свет в электричество для удовлетворения своих потребностей в энергии. (Источник: Эван-Амос, Wikimedia Commons)

4. Сколько ватт проходит через него фонарик с 6,00 × 10 2 за 0,500 ч использования, если его напряжение составляет 3,00 В?

5. Найдите мощность, рассеиваемую в каждом из этих удлинителей: (a) удлинительный шнур с сопротивлением 0,0600 Ом, через который 5.00 А течет; (б) более дешевый шнур с более тонким проводом и сопротивлением 0,300 Ом.

6. Убедитесь, что единицами измерения вольт-ампер являются ватты, как следует из уравнения P = IV .

7. Покажите, что единицы 1V 2 / Ω = 1W, как следует из уравнения P = V 2 / R .

8. Покажите, что единицы 1 A 2 Ω = 1 Вт, как следует из уравнения P = I 2 R .

9. Проверьте эквивалент единиц энергии: 1 кВт ч = 3,60 × 10 6 Дж.

10. Электроны в рентгеновской трубке ускоряются до 1,00 × 10 2 кВ и направляются к цели для получения рентгеновских лучей. Вычислите мощность электронного луча в этой трубке, если она имеет ток 15,0 мА.

11. Электрический водонагреватель потребляет 5,00 кВт на 2,00 часа в сутки. Какова стоимость его эксплуатации в течение одного года, если электроэнергия стоит 12,0 центов / кВт · ч? См. Рисунок 3.

Рисунок 3. Водонагреватель электрический по запросу. Тепло в воду подается только при необходимости. (кредит: aviddavid, Flickr)

12. Сколько электроэнергии необходимо для тостера с тостером мощностью 1200 Вт (время приготовления = 1 минута)? Сколько это стоит при 9,0 цента / кВт · ч?

13. Какова будет максимальная стоимость КЛЛ, если общая стоимость (капиталовложения плюс эксплуатация) будет одинаковой как для КЛЛ, так и для ламп накаливания мощностью 60 Вт? Предположим, что стоимость лампы накаливания составляет 25 центов, а электричество стоит 10 центов / кВтч.Рассчитайте стоимость 1000 часов, как в примере с КЛЛ по рентабельности.

14. Некоторые модели старых автомобилей имеют электрическую систему напряжением 6,00 В. а) Каково сопротивление горячему свету у фары мощностью 30,0 Вт в такой машине? б) Какой ток течет через него?

15. Щелочные батареи имеют то преимущество, что они выдают постоянное напряжение почти до конца своего срока службы. Как долго щелочная батарея с номиналом 1,00 А · ч и 1,58 В будет поддерживать горение лампы фонарика мощностью 1,00 Вт?

16.Прижигатель, используемый для остановки кровотечения в хирургии, выдает 2,00 мА при 15,0 кВ. а) Какова его выходная мощность? б) Какое сопротивление пути?

17. В среднем телевизор работает 6 часов в день. Оцените ежегодные затраты на электроэнергию для работы 100 миллионов телевизоров, предполагая, что их потребляемая мощность составляет в среднем 150 Вт, а стоимость электроэнергии составляет в среднем 12,0 центов / кВт · ч.

18. Старая лампочка потребляет всего 50,0 Вт, а не 60,0 Вт из-за истончения ее нити за счет испарения.Во сколько раз уменьшается его диаметр при условии равномерного утонения по длине? Не обращайте внимания на любые эффекты, вызванные перепадами температур.

Медная проволока калибра 19. 00 имеет диаметр 9,266 мм. Вычислите потери мощности в километре такого провода, когда он пропускает 1,00 × 10 2 A.

Холодные испарители пропускают ток через воду, испаряя ее при небольшом повышении температуры. Одно такое домашнее устройство рассчитано на 3,50 А и использует 120 В переменного тока с эффективностью 95,0%.а) Какова скорость испарения в граммах в минуту? (b) Сколько воды нужно налить в испаритель за 8 часов работы в ночное время? (См. Рисунок 4.)

Рис. 4. Этот холодный испаритель пропускает ток непосредственно через воду, испаряя ее напрямую с относительно небольшим повышением температуры.

21. Integrated Concepts (a) Какая энергия рассеивается разрядом молнии с током 20 000 А, напряжением 1,00 × 10 2 МВ и длиной 1.00 мс? (б) Какую массу древесного сока можно было бы поднять с 18ºC до точки кипения, а затем испарить за счет этой энергии, если предположить, что сок имеет те же тепловые характеристики, что и вода?

22. Integrated Concepts Какой ток должен вырабатывать подогреватель бутылочек на 12,0 В, чтобы нагреть 75,0 г стекла, 250 г детской смеси и 3,00 × 10 2 алюминия от 20 ° C до 90º за 5,00 мин?

23. Integrated Concepts Сколько времени требуется хирургическому прижигателю, чтобы поднять температуру на 1.00 г ткани от 37º до 100, а затем закипятите 0,500 г воды, если она выдает 2,00 мА при 15,0 кВ? Не обращайте внимания на передачу тепла в окружающую среду.

24. Integrated Concepts Гидроэлектрические генераторы (см. Рисунок 5) на плотине Гувера вырабатывают максимальный ток 8,00 × 10 3 A при 250 кВ. а) Какова выходная мощность? (b) Вода, питающая генераторы, входит и покидает систему с низкой скоростью (таким образом, ее кинетическая энергия не изменяется), но теряет 160 м в высоте.Сколько кубических метров в секунду необходимо при КПД 85,0%?

Рисунок 5. Гидроэлектрические генераторы на плотине Гувера. (кредит: Джон Салливан)

25. Integrated Concepts (a) Исходя из 95,0% эффективности преобразования электроэнергии двигателем, какой ток должны обеспечивать аккумуляторные батареи на 12,0 В 750-килограммового электромобиля: отдых до 25,0 м / с за 1,00 мин? (b) Подняться на холм высотой 2,00 × 10 2 м за 2,00 мин при постоянной 25.Скорость 0 м / с при приложении силы 5,00 × 10 2 Н для преодоления сопротивления воздуха и трения? (c) Двигаться с постоянной скоростью 25,0 м / с, прилагая силу 5,00 × 10 2 Н для преодоления сопротивления воздуха и трения? См. Рисунок 6.

Рис. 6. Электромобиль REVAi заряжается на одной из улиц Лондона. (кредит: Фрэнк Хебберт)

26. Integrated Concepts Пригородный легкорельсовый поезд потребляет 630 А постоянного тока напряжением 650 В при ускорении.а) Какова его мощность в киловаттах? (b) Сколько времени нужно, чтобы достичь скорости 20,0 м / с, начиная с состояния покоя, если его загруженная масса составляет 5,30 × 10 4 кг, предполагая эффективность 95,0% и постоянную мощность? (c) Найдите его среднее ускорение. (г) Обсудите, как ускорение, которое вы обнаружили для легкорельсового поезда, сравнивается с тем, что может быть типичным для автомобиля.

27. Integrated Concepts (a) Линия электропередачи из алюминия имеет сопротивление 0,0580 Ом / км. Какова его масса на километр? б) Какова масса на километр медной линии с таким же сопротивлением? Более низкое сопротивление сократит время нагрева.Обсудите практические ограничения ускорения нагрева за счет снижения сопротивления.

28. Integrated Concepts (a) Погружной нагреватель, работающий на 120 В, может повысить температуру 1,00 × 10 2 -г алюминиевого стакана, содержащего 350 г воды, с 20 ° C до 95 ° C за 2,00 мин. Найдите его сопротивление, предполагая, что оно постоянно в процессе. (b) Более низкое сопротивление сократит время нагрева. Обсудите практические ограничения ускорения нагрева за счет снижения сопротивления.

29. Integrated Concepts (a) Какова стоимость нагрева гидромассажной ванны, содержащей 1500 кг воды, от 10 ° C до 40 ° C, исходя из эффективности 75,0% с учетом передачи тепла в окружающую среду? Стоимость электроэнергии 9 центов / кВт⋅ч. (b) Какой ток потреблял электрический нагреватель переменного тока 220 В, если на это потребовалось 4 часа?

30 . Необоснованные результаты (a) Какой ток необходим для передачи 1,00 × 10 2 МВт мощности при 480 В? (b) Какая мощность рассеивается линиями передачи, если они имеют коэффициент 1.00 — сопротивление Ом? (c) Что неразумного в этом результате? (d) Какие предположения необоснованны или какие посылки несовместимы?

31. Необоснованные результаты (a) Какой ток необходим для передачи 1,00 × 10 2 МВт мощности при 10,0 кВ? (b) Найдите сопротивление 1,00 км провода, которое вызовет потерю мощности 0,0100%. (c) Каков диаметр медного провода длиной 1,00 км, имеющего такое сопротивление? (г) Что необоснованного в этих результатах? (e) Какие предположения необоснованны или какие посылки несовместимы?

32.Создайте свою проблему Рассмотрим электрический погружной нагреватель, используемый для нагрева чашки воды для приготовления чая. Постройте задачу, в которой вы рассчитываете необходимое сопротивление нагревателя, чтобы он увеличивал температуру воды и чашки за разумное время. Также рассчитайте стоимость электроэнергии, используемой в вашем технологическом процессе. Среди факторов, которые необходимо учитывать, — это используемое напряжение, задействованные массы и теплоемкость, тепловые потери и время, в течение которого происходит нагрев.Ваш инструктор может пожелать, чтобы вы рассмотрели тепловой предохранительный выключатель (возможно, биметаллический), который остановит процесс до того, как в погружном блоке будут достигнуты опасные температуры.

Глоссарий

электрическая мощность:
скорость, с которой электрическая энергия подается источником или рассеивается устройством; это произведение тока на напряжение

Избранные решения проблем и упражнения

1. 2,00 × 10 12 Вт

5.{6} \ text {J} \\ [/ latex]

11. 438 $ / год

13. $ 6.25

15. 1.58 ч

17. 3,94 миллиарда долларов в год

19. 25,5 Вт

21. (а) 2,00 × 10 9 Дж (б) 769 кг

23. 45.0 с

25. (а) 343 A (б) 2,17 × 10 3 A (в) 1,10 × 10 3 A

27. (а) 1,23 × 10 3 кг (б) 2,64 × 10 3 кг

29. (a) 2,08 × 10 5 A
(b) 4,33 × 10 4 МВт
(c) Линии передачи рассеивают больше мощности, чем они должны передавать.
(d) Напряжение 480 В неоправданно низкое для напряжения передачи. В линиях передачи на большие расстояния поддерживается гораздо более высокое напряжение (часто сотни киловольт), чтобы уменьшить потери мощности.

Что такое амперы, ватты, вольт и омы?

В электрической системе увеличение тока или напряжения приведет к увеличению мощности. Допустим, у вас есть система с 6-вольтовой лампочкой, подключенной к 6-вольтовой батарее. Выходная мощность лампочки составляет 100 Вт.Используя уравнение I = P / V , мы можем рассчитать, какой ток в амперах потребуется, чтобы получить 100 Вт от этой 6-вольтовой лампы.

Вы знаете, что P = 100 Вт, а V = 6 В. Итак, вы можете изменить уравнение, чтобы найти I и подставить числа.

I = 100 Вт / 6 В = 16,67 А

Что произойдет, если использовать 12-вольтовую батарею и 12-вольтовую лампочку, чтобы получить мощность 100 Вт?

I = 100 Вт / 12 В = 8,33 ампера

Итак, последняя система вырабатывает ту же мощность, но с половиной тока.Преимущество заключается в использовании меньшего тока для получения того же количества энергии. Сопротивление в электрических проводах потребляет мощность, а потребляемая мощность увеличивается по мере увеличения тока, проходящего по проводам. Вы можете увидеть, как это происходит, немного изменив два уравнения. Что вам нужно, так это уравнение мощности с точки зрения сопротивления и тока. Давайте изменим первое уравнение:

I = V / R можно переформулировать как V = I * R

Теперь вы можете подставить уравнение для V в другое уравнение:

P = V * I, подставив V, мы получим P = I * R * I, или P = I 2 * R

Это уравнение говорит вам о том, что мощность, потребляемая проводами, увеличивается, если сопротивление проводов увеличивается (например, если провода становятся меньше или изготовлены из менее проводящего материала).Но он резко возрастает, если ток, протекающий по проводам, увеличивается. Таким образом, использование более высокого напряжения для уменьшения тока может сделать электрические системы более эффективными. КПД электродвигателей также повышается при более высоких напряжениях.

Это повышение эффективности побудило автомобильную промышленность в 1990-х годах задуматься о переходе с 12-вольтовых электрических систем на 42-вольтовые системы. По мере того, как все больше автомобилей поставлялись с электрическими удобствами — видеодисплеями, обогревателями сидений, «умным» климат-контролем, им требовались толстые пучки проводов для обеспечения достаточного тока.Переключение на систему с более высоким напряжением обеспечит большую мощность при более тонкой проводке.

Переключения не произошло, потому что автопроизводители смогли повысить эффективность с помощью цифровых технологий и более эффективных электрических насосов на 12 вольт. Но в некоторых новых моделях используются гибридные системы с отдельным 48-вольтовым генератором для питания расширенных функций, таких как отключение на холостом ходу, при одновременном повышении общей эффективности системы.

Чтобы узнать больше об электричестве и связанных темах, воспользуйтесь ссылками на следующей странице.

Закон Ома

Закон

Ома показывает линейную зависимость между напряжением и током в электрической цепи.

Падение напряжения и сопротивление резистора определяют протекание постоянного тока через резистор.

Используя аналогию с потоком воды, мы можем представить электрический ток как ток воды через трубу, резистор как тонкую трубу, которая ограничивает поток воды, напряжение как разница высот воды, которая обеспечивает течение воды.

Формула закона Ома

Ток I резистора в амперах (A) равен току резистора напряжение V в вольтах (В), деленное на сопротивление R в омах (Ом):

В — падение напряжения на резисторе, измеренное в вольтах (В).В некоторых случаях в законе Ома для обозначения напряжения используется буква E . E обозначает электродвижущую силу.

I — электрический ток, протекающий через резистор, измеряемый в амперах (A)

R — сопротивление резистора, измеренное в Ом (Ом)

Расчет напряжения

Зная ток и сопротивление, мы можем рассчитать напряжение.

Напряжение V в вольтах (В) равно току I в амперах (А), умноженному на сопротивление R в омах (Ом):

Расчет сопротивления

Зная напряжение и ток, мы можем рассчитать сопротивление.

Сопротивление R в омах (Ом) равно напряжению V в вольтах (В), деленному на ток I в амперах (A):

Поскольку ток задается значениями напряжения и сопротивления, формула закона Ома может показать, что:

  • Если увеличивать напряжение, ток увеличится.
  • Если мы увеличим сопротивление, ток уменьшится.
Пример # 1

Найдите ток электрической цепи с сопротивлением 50 Ом и напряжением питания 5 Вольт.

Решение:

В = 5 В

R = 50 Ом

I = В / R = 5 В / 50 Ом = 0,1 А = 100 мА

Пример # 2

Найдите сопротивление электрической цепи, имеющей напряжение питания 10 В и ток 5 мА.

Решение:

В = 10 В

I = 5 мА = 0,005 А

R = В / I = 10 В / 0,005 A = 2000 Ом = 2 кОм

Закон Ома для цепи переменного тока

Ток нагрузки I в амперах (A) равен напряжению нагрузки V Z = V в вольтах (В), деленному на полное сопротивление Z в омах (Ом):

В — падение напряжения на нагрузке, измеренное в вольтах (В)

I — электрический ток, измеряемый в амперах (A)

Z — полное сопротивление нагрузки, измеренное в Ом (Ом)

Пример # 3

Найдите ток в цепи переменного тока с напряжением питания 110 В ± 70 ° и нагрузкой 0.5кОм∟20 °.

Решение:

В = 110 В 70 °

Z = 0,5 кОм∟20 ° = 500 Ом∟20 °

I = В / Z = 110 В 70 ° / 500 Ом 20 ° = (110 В / 500 Ом) ∟ (70 ° -20 °) = 0,22 А 50 °

Калькулятор закона Ома (краткая форма)

Калькулятор закона

Ома: вычисляет соотношение между напряжением, током и сопротивлением.

Введите 2 значений, чтобы получить третье значение, и нажмите кнопку Рассчитать :

Калькулятор закона Ома II ►


См. Также

Закон Ома и соотношение V-I-R

В физике есть определенные формулы, которые настолько мощны и распространены, что достигают уровня общеизвестных знаний.Студент, изучающий физику, записывал такие формулы столько раз, что запоминал их, даже не пытаясь. Безусловно, для профессионалов в этой области такие формулы настолько важны, что остаются в их сознании. В области современной физики E = m • c 2 . В области ньютоновской механики существует F net = m • a. В области волновой механики v = f • λ. А в области текущего электричества ΔV = I • R.

Преобладающим уравнением, которое пронизывает изучение электрических цепей, является уравнение

ΔV = I • R

Другими словами, разность электрических потенциалов между двумя точками в цепи ( ΔV ) эквивалентна произведению тока между этими двумя точками ( I ) и общего сопротивления всех электрических устройств, присутствующих между этими двумя точками ( R ).В остальной части этого раздела Физического класса это уравнение станет самым распространенным уравнением, которое мы видим. Это уравнение, часто называемое уравнением закона Ома , является мощным предсказателем взаимосвязи между разностью потенциалов, током и сопротивлением.

Закон Ома как предсказатель тока

Уравнение закона Ома можно переформулировать и выразить как

В качестве уравнения это служит алгебраическим рецептом для вычисления тока, если известны разность электрических потенциалов и сопротивление.Тем не менее, хотя это уравнение служит мощным рецептом решения проблем, это гораздо больше. Это уравнение указывает две переменные, которые могут повлиять на величину тока в цепи. Ток в цепи прямо пропорционален разности электрических потенциалов, приложенной к ее концам, и обратно пропорционален общему сопротивлению внешней цепи. Чем больше напряжение аккумулятора (то есть разность электрических потенциалов), тем больше ток. И чем больше сопротивление, тем меньше ток.Заряд идет с наибольшей скоростью, когда напряжение батареи увеличивается, а сопротивление уменьшается. Фактически, двукратное увеличение напряжения батареи привело бы к двукратному увеличению тока (если все остальные факторы остаются равными). А увеличение сопротивления нагрузки в два раза приведет к уменьшению тока в два раза до половины его первоначального значения.

Таблица ниже иллюстрирует эту взаимосвязь как качественно, так и количественно для нескольких цепей с различными напряжениями и сопротивлением батарей.


Строки 1, 2 и 3 показывают, что удвоение и утроение напряжения батареи приводит к удвоению и утроению тока в цепи. Сравнение строк 1 и 4 или строк 2 и 5 показывает, что удвоение общего сопротивления служит для уменьшения вдвое тока в цепи.

Поскольку на ток в цепи влияет сопротивление, в цепях электроприборов часто используются резисторы, чтобы влиять на величину тока, присутствующего в ее различных компонентах.Увеличивая или уменьшая величину сопротивления в конкретной ветви схемы, производитель может увеличивать или уменьшать величину тока в этой ветви . Кухонные приборы, такие как электрические миксеры и переключатели света, работают, изменяя ток в нагрузке, увеличивая или уменьшая сопротивление цепи. Нажатие различных кнопок на электрическом микшере может изменить режим с микширования на взбивание, уменьшив сопротивление и позволив большему току присутствовать в миксере.Точно так же поворот ручки регулятора яркости может увеличить сопротивление его встроенного резистора и, таким образом, уменьшить ток.

На схеме ниже изображена пара цепей, содержащих источник напряжения (аккумуляторная батарея), резистор (лампочка) и амперметр (для измерения тока). В какой цепи у лампочки наибольшее сопротивление? Нажмите кнопку «Посмотреть ответ», чтобы убедиться, что вы правы.


Уравнение закона Ома часто исследуется в физических лабораториях с использованием резистора, аккумуляторной батареи, амперметра и вольтметра.Амперметр — это устройство, используемое для измерения силы тока в заданном месте. Вольтметр — это устройство, оснащенное датчиками, которых можно прикоснуться к двум точкам цепи, чтобы определить разность электрических потенциалов в этих местах. Изменяя количество ячеек в аккумуляторной батарее, можно изменять разность электрических потенциалов во внешней цепи. Вольтметр может использоваться для определения этой разности потенциалов, а амперметр может использоваться для определения тока, связанного с этим ΔV.К батарейному блоку можно добавить батарею, и процесс можно повторить несколько раз, чтобы получить набор данных I-ΔV. График зависимости I от ΔV даст линию с крутизной, эквивалентной обратной величине сопротивления резистора. Это значение можно сравнить с заявленным производителем значением, чтобы определить точность лабораторных данных и справедливость уравнения закона Ома.

Величины, символы, уравнения и единицы!

Тенденция уделять внимание единицам — неотъемлемая черта любого хорошего студента-физика.Многие трудности, связанные с решением проблем, могут быть связаны с тем, что не уделили внимание подразделениям. Поскольку все больше и больше электрических величин и их соответствующие метрические единицы вводятся в этом разделе учебного пособия «Физический класс», становится все более важным организовать информацию в своей голове. В таблице ниже перечислены некоторые из введенных на данный момент количеств. Для каждой величины также указаны символ, уравнение и соответствующие метрические единицы.Было бы разумно часто обращаться к этому списку или даже делать свою копию и добавлять к ней по мере развития модуля. Некоторые студенты считают полезным составить пятый столбец, в котором приводится определение каждой величины.

Кол-во Символ Уравнение (а) Стандартная метрическая единица Другие единицы
Разность потенциалов

(г.к.а. напряжение)

ΔV ΔV = ΔPE / Q

ΔV = I • R

Вольт (В) J / C
Текущий я I = Q / т

I = ΔV / R

Амперы (А) Усилитель или К / с

или В / Ом

Мощность п P = ΔPE / т

(еще впереди)

Ватт (Вт) Дж / с
Сопротивление р R = ρ • L / A

R = ΔV / I

Ом (Ом) В / А
Энергия E или ΔPE ΔPE = ΔV • Q

ΔPE = P • t

Джоуль (Дж) V • C или

Вт • с

(Обратите внимание, что символ C представляет собой кулоны.)

В следующем разделе Урока 3 мы еще раз рассмотрим количественную мощность. Новое уравнение мощности будет введено путем объединения двух (или более) уравнений в приведенной выше таблице.

Мы хотели бы предложить … Зачем просто читать об этом и когда можно с этим взаимодействовать? Взаимодействовать — это именно то, что вы делаете, когда используете одно из интерактивных материалов The Physics Classroom.Мы хотели бы предложить вам совместить чтение этой страницы с использованием нашего интерактивного средства построения цепей постоянного тока. Вы можете найти его в разделе Physics Interactives на нашем сайте. Построитель цепей постоянного тока предоставляет учащемуся набор для построения виртуальных цепей. Легко перетащите источник напряжения, резисторы и провода на рабочее место. Соедините их, и у вас будет схема. Добавьте амперметр для измерения тока и используйте датчики напряжения для определения падения напряжения. Это так просто. И не нужно беспокоиться о поражении электрическим током (если, конечно, вы не читаете это в ванной).


Проверьте свое понимание

1. Что из перечисленного ниже приведет к уменьшению тока в электрической цепи? Выберите все, что подходит.

а. уменьшить напряжение

г. уменьшить сопротивление

г. увеличить напряжение

г.увеличить сопротивление

2. Определенная электрическая цепь содержит батарею из трех элементов, провода и лампочку. Что из перечисленного может привести к тому, что лампа будет светить менее ярко? Выберите все, что подходит.

а. увеличить напряжение АКБ (добавить еще одну ячейку)

г. уменьшить напряжение аккумулятора (удалить элемент)

г.уменьшить сопротивление цепи

г. увеличить сопротивление цепи

3. Вероятно, вас предупредили, чтобы вы не прикасались к электроприборам или даже к электрическим розеткам мокрыми руками. Такой контакт более опасен, когда ваши руки мокрые (а не сухие), потому что мокрые руки вызывают ____.

а.напряжение цепи должно быть выше

г. напряжение цепи должно быть ниже

г. ваше сопротивление будет выше

г. ваше сопротивление должно быть ниже

e. ток через тебя будет ниже

4. Если бы сопротивление цепи было утроено, то ток в цепи был бы ____.

а. треть от

г. втрое больше

г. без изменений

г. … ерунда! Сделать такой прогноз невозможно.

5. Если напряжение в цепи увеличить в четыре раза, то ток в цепи будет ____.

а.четверть от

г. в четыре раза больше

г. без изменений

г. … ерунда! Сделать такой прогноз невозможно.

6. Схема соединена с блоком питания, резистором и амперметром (для измерения тока). Амперметр показывает значение тока 24 мА (миллиАмпер). Определите новый ток, если напряжение источника питания было…

а. … увеличилось в 2 раза, а сопротивление осталось постоянным.

г. … увеличился в 3 раза, а сопротивление осталось постоянным.

г. … уменьшилось в 2 раза, а сопротивление осталось постоянным.

г. … оставалось постоянным, а сопротивление увеличивалось в 2 раза.

e. … оставалось постоянным, а сопротивление увеличивалось в 4.

ф…. оставалось постоянным, а сопротивление уменьшалось в 2 раза.

г. … увеличилось в 2 раза, а сопротивление увеличилось в 2 раза.

ч. … увеличилось в 3 раза, а сопротивление уменьшилось в 2 раза.

и. … уменьшилось в 2 раза, а сопротивление увеличилось в 2 раза.

7.Используйте уравнение закона Ома, чтобы дать числовые ответы на следующие вопросы:

а. Электрическое устройство с сопротивлением 3,0 Ом позволит протекать через него току 4,0 А, если на устройстве наблюдается падение напряжения ________ Вольт.

г. Когда на электрический нагреватель подается напряжение 120 В, через нагреватель будет протекать ток 10,0 А, если сопротивление составляет ________ Ом.

г. Фонарик с питанием от 3 вольт и лампочкой с сопротивлением 60 Ом будет иметь ток ________ ампер.

8. Используйте уравнение закона Ома для определения недостающих значений в следующих схемах.

9.

Добавить комментарий

Ваш адрес email не будет опубликован. Обязательные поля помечены *